You are on page 1of 95

CLASSIFICATION OF PROPERTY

Immovable vs. Movable Properties

Laurel vs. Abrogar


G.R. NO. 155076
Jan. 13, 2009

Facts: Baynet was charged with engaging in International Simple Resale (ISR) or the
unauthorized routing and completing of international long distance calls using lines, cables,
antennae, and/or air wave frequency and connecting these calls directly to the local or
domestic exchange facilities of the country destined.

PLDT alleges that the international phone calls which are electric currents or sets of electric
impulses transmitted through a medium, and carry a pattern representing the human voice to a
receiver, are personal properties which may be the subject of theft. Art 416 (3) deems forces of
nature which are brought under the control by science, are personal property.

Laurel, one of the officers of Baynet, claims that a telephone call is a conversation on the
phone or a communication carried out using the telephone. It is not synonymous to electric
currents or impulses. Hence, it may not be considered as personal property susceptible of
appropriation. Laurel claims that the analogy between generated electricity and telephone calls
is misplaced. PLDT does not produce or generate telephone calls. It only provides the facilities
or services for the transmission and switching of the calls. He also insists that business is not a
personal property. It is not the business that is protected but the right to carry a business. This
right is what is considered as property. Since the services of PLDT cannot be considered as
property, the same may not be the subject of theft.

Issue: Whether or not the business of providing telecommunication or telephone service are
personal properties capable of appropriation and can be objects of theft.

Ruling: Yes, interest in business was declared to be personal property since it is capable of
appropriation and not included in the enumeration of real properties. Art. 414 provides that all
things which are or may be the object of appropriation are considered either real property or
personal property.Business is likewise not enumerated as personal property under the Civil
Code. Just like interest in business, however, it may be appropriated. Business should also be
classified as personal property. Since it is not included in the exclusive enumeration of real
properties under Art. 415, it is therefore personal property.

In making the international phone calls, the human voice is converted into electrical impulses or
electric current which are transmitted to the part called. A telephone call, therefore, is
electrical energy. Intagnible property such as electrical energy is capable of appropriation
because it may be taken and carried away. Electricity is personal property under Art. 416 (3)
which enumerates “forces of nature which are brought under control by science".
Kinds of Immovable Properties

Bicerra vs. Teneza


G.R. No. L-16218
November 29, 1962

Facts: The complaint alleges that appellants were the owners of the house, worth P200.00,
built on and owned by them and situated in the said municipality Lagangilang; that sometime in
January 1957 their house was forcibly demolished by the appellees, claiming they were owners
thereof; that the materials of the house, after it was dismantled, were placed in the custody of
the barrio lieutenant of the place; and that as a result of appellee’s refusal to restore the house
or to deliver the material appellants the latter have suffered actual damage, moral and
consequential damages. The relief prayed for is that "the plaintiffs be declared the owners of
the house in question and/or the materials that resulted in its dismantling be restored.

Issue: Whether or not the action involves a title to a real property.

Ruling: No, the action does not involve a title to real property. A house is classified as
immovable property by reason of its adherence to the soil on which it is built (Art. 415, par. 1,
Civil Code). This classification holds true regardless of the fact that the house may be situated
on land belonging to a different owner. But once the house is demolished, as in this case, it
ceases to exist as such and hence its character as an immovable likewise ceases.
Punzalan, Jr. v. Vda. De Lacsamana
G.R. No. L-55729
March 28, 1983

Facts: Antonio Punsalan, Jr., was the former registered owner of a parcel of land. In 1963,
petitioner mortgaged said land to respondent PNB, but for failure to pay said amount, the
property was foreclosed. Respondent PNB was the highest bidder in said foreclosure
proceedings. However, the bank secured title thereto only on December 14, 1977.

On a relevant date, while Punsalan was still the possessor of the land, it secured a
permit for the construction of a warehouse which he eventually leased.

A Deed of Sale was executed between respondent PNB and respondent Lacsamana over the
property. This contract was amended, particularly to include in the sale, the building and
improvement thereon. By virtue of said instruments, respondent - Lacsamana secured title over
the property in her name (as well as separate tax declarations for the land and building.

Petitioner commenced suit for "Annulment of Deed of Sale with Damages" against herein
respondents PNB and Lacsamana essentially impugning the validity of the sale of the building
which was not owned by the bank.

Issue: Whether or not the warehouse is an immovable and must be tried in the province where
the property lies.

Ruling: Warehouse claimed to be owned by petitioner is an immovable or real property.


Buildings are always immovable under the Civil Code. A building treated separately from the
land on which it is stood is immovable property and the mere fact that the parties to a contract
seem to have dealt with it separate and apart from the land on which it stood did not change its
character as immovable property.
Tsai vs. CA
G.R. No. 120098
October 2, 2001

Facts: Evertex secured a loan from PBC, guaranteed by real estate and chattel mortgage over a
parcel of land where the factory stands, and the chattels located therein, as included in a
schedule attached to the mortgage contract. Another loan was obtained secured by a chattel
mortgage over properties with similar descriptions listed in the first schedule. After the
execution of the second mortgage, Evertex purchased machineries and equipment.

Due to business reverses, Evertex filed for insolvency proceedings. It failed to pay its obligation
and thus, PBC initiated extrajudicial foreclosure of the mortgages where PBC was the highest
bidder in the public auctions, making it the owner of the properties. It then leased the factory
premises to Tsai.

Subsequently, Evertex sought the annulment of the sale and conveyance of the properties to
PBC as it was allegedly a violation of the insolvency law.

Issue: Whether or not the immovable properties in question can be entered into a chattel
mortgage.

Ruling: Yes, an immovable may be considered a personal property if there is a stipulation as


when it is used as security in the payment of an obligation where a chattel mortgage is
executed over it, as in the case at bar. While it is true that the controverted properties appear
to be immobile, a perusal of the contract of real estate mortgage and chattel mortgage by the
parties gives a contrary indication. Both the trial and appellate courts show that the intention
was to treat the machineries as movables or personal property.

Assuming that the properties were considered immovables, nothing detracts the parties from
treating it as chattels to secure an obligation under the principle of estoppel.
Caltex Philippines vs. CBAA
G.R. No. L-50466
May 31, 1982

Facts: The petitioner Caltex loaned machines and equipment to gas station operators under an
appropriate lease agreement. On the lease contract stipulated that upon demand, the
operators shall return to the Caltex the machines and equipment in good condition as when
received, ordinary wear and tear expected. The lessor of the land, where the gas station is
located, does not become the owner of the machines and equipment installed therein. Caltex
retains the ownership thereof during the term of lease.
The city assessor of pasay characterized said items of gas station machines and equipment as
taxable realty. However, the city board of tax appeals ruled that they are personalty. The board
held that the machinery are real property within the meaning of sec. 3(k) and 38 of the real
property tax code.

Issue: The machinery and equipment permanently affixed by Caltex to its gas station and
pavement is considered as real property and subject to real property tax.

Held: Yes, the court ruled that the machinery and equipment were subject to real property tax.
The court cited the provision in tax code that sec. 2 of the assessment law provides that the
realty tax is due on real property, including land, buildings, machinery and other improvements
not specifically exempted in sec. 3 thereof.

Sec. 3 of the real property tax code provides that the following definitions:
k.) improvements- a valuable addition made to property or an amelioration in its condition
more than mere repairs or replacement of waste… intended to enhance its value, beauty, or
utility
m.) machinery- machines, mechanical contrivances, instruments, appliances, and apparatus
attached to the real estate includes the physical facilities available for production installation
and appurtenant service facilities.
Meralco Securities vs. CBAA
G.R. No. L46245
May 31, 1982

Facts: Petitioner Meralco installed a pipeline system from Manila to Batangas. The provincial
assessor of Laguna treated the pipeline as real property and imposed a real property tax over
the pipeline system. The petitioner questioned the tax impose to the pipeline system that the
thing is considered as personal property because it can be change from place to place.

Issue: Whether or not the pipeline system is considered as real property.

Held: The Supreme Court held that the pipeline system is considered as real property.
Rights as Property

Metropolitan Bank and Trust Co. vs. Alejo

Facts: Spouses Raul and Cristina Acampado obtained loans from MBTC in the amount of
P5,000.00. As security of their payment spouses executed in favor of the bank a real estate
mortgage over a parcel of land registered in their names. Sy Tan Sy filed a complaint for the
declaration of nullity of the TCT issued to the spouses in the RTC of Valenzuela. The spouses
defaulted on the payment of their so the collateral was foreclosed.
The bank submitted the highest and winning bid. The certificate of sale was issued in their
favor. When the bank were about to get their TCT a declaration of nullity was informed of the
existence of the TCT in the aforementioned civil case declaring that spouses Acampado’s TCT
was null and void.

Issue: Whether or not a petition for annulment of judgment was the proper remedy available
to the bank.

Held: The court ruled that the petition for annulment was the proper remedy available to the
bank. It precisely alleged that Sy concealed the case by excluding the petitioner as a defendant
to the civil case even he was an indispensable party to the case. This deprived the bank of its
duly registered property right without due process of law. The allegation of fraud may be the
basis for annulling a judgment.
Constitutional basis of State Ownership – Jura Regalia

Chavez vs. Public Estate Authority


415 scra 403
July 9, 2002

Facts: President Marcos through a presidential decree created PEA, which was tasked with the
development, improvement, and acquisition, lease and sale of all kinds of lands. Then, the
president also transferred to PEA the foreshore and offshore lands of Manila Bay under the
Manila-Cavite Coastal Road and Reclamation Project. PEA entered into a JVA with AMARI for
the development of the freedom Islands. These two entered into a joint venture in the absence
of any public bidding.

A privilege speech was given by senator president maceda denouncing the JVA as he
grandmother of all scams. An investigation was conducted and it was concluded that the lands
that PEA was conveying to AMARI were lands of the public domain, the certificates of title over
the freedom islands were void and the JVA itself was illegal. Petitioner contended that the
government stands to lose billions by the conveyance or sale of the reclaimed areas to AMARI.
He also asked for the full disclosure of the renegotiations happening between the parties.

Issue: Whether or not the stipulations in the amended JVA for the transfer to AMARI of the
lands, reclaimed or to be reclaimed, violate the Constitution.

Ruling: The foreshore and submerged areas of the Manila Bay are part of the lands of the
public domain, waters and other natural resources and consequently owned by the state. As
such, foreshore and submerged areas shall not be alienable unless they are classified as
agricultural lands of the public domain. The mere reclamation of these areas by the PEA does
not convert these inalienable natural resources of the state into alienable and disposable lands
of the public domain. There must be a law or presidential proclamation officially classifying
these reclaimed lands as alienable and disposable if the law has reserved them for some public
or quasi-public use.

The amended JVA covers a reclamation area of 750 hectares. Only 157.84 hectares of the 750
hectare reclamation project have been reclaimed, and the rest of the area are still submerged
areas forming part of Manila Bay. Further, it is provided that AMARI will reimburse the actual
costs in reclaiming the areas of land and it will shoulder the other reclamation costs to be
incurred.
Republic of the Philippines vs. Santos
G.R. No. 160453
November 12, 2012

Facts: Alleging continuous and adverse possession of more than ten years, respondent Arcadio
Ivan A. Santos III (Arcadio Ivan) applied on March 7, 1997 for the registration of Lot 4998-B (the
property) in the Regional Trial Court (RTC) in Parafiaque City. The property, which had an area
of 1,045 square meters, more or less, was located in Barangay San Dionisio, Paraque City, and
was bounded in the Northeast by Lot 4079 belonging to respondent Arcadio C. Santos, Jr.
(Arcadio, Jr.), in the Southeast by the Paraque River, in the Southwest by an abandoned road,
and in the Northwest by Lot 4998-A also owned by Arcadio Ivan. On May 21, 1998, Arcadio Ivan
amended his application for land registration to include Arcadio, Jr. as his co-applicant because
of the latters co-ownership of the property. He alleged that the property had been formed
through accretion and had been in their joint open, notorious, public, continuous and adverse
possession for more than 30 years.

Issue: Whether or not the subject parcel land maybe acquired through the process of accretion.

Held: No. Accretion is the process whereby the soil is deposited along the banks of rivers. The
deposit of soil, to be considered accretion, must be: (a) gradual and imperceptible; (b) made
through the effects of the current of the water; and (c) taking place on land adjacent to the
banks of rivers.
The RTC and the CA grossly erred in treating the dried-up river bed as an accretion that became
respondents property pursuant to Article 457 of the Civil Code. That land was definitely not an
accretion. The process of drying up of a river to form dry land involved the recession of the
water level from the river banks, and the dried-up land did not equate to accretion, which was
the gradual and imperceptible deposition of soil on the river banks through the effects of the
current. In accretion, the water level did not recede and was more or less maintained. Hence,
respondents as the riparian owners had no legal right to claim ownership of Lot 4998-B.
Considering that the clear and categorical language of Article 457 of the Civil Code has confined
the provision only to accretion, we should apply the provision as its clear and categorical
language tells us to. Axiomatic it is, indeed, that where the language of the law is clear and
categorical, there is no room for interpretation; there is only room for application. The first and
fundamental duty of courts is then to apply the law.
PUBLIC OWNERSHIP VS STATE OWNERSHIP

Public Service vs Public Use

Teofilo C. Villarico vs. Vivencio Sarmiento


G.R. No. 136438
November 11, 2004

Facts: Teofilo C. Villarico is the owner of a lot in La Huerta, Parañaque City, Metro Manila with
an area of 66 square meters and covered by Transfer Certificate of Title (T.C.T.) No. 95453
issued by the Registry of Deeds, same city.

Villarico’s lot is separated from the Ninoy Aquino Avenue (highway) by a strip of land belonging
to the government. As this highway was elevated by 4 meters and therefore higher than the
adjoining areas, the DPWH constructed stairways at several portions of this strip of public land
to enable the people to have access to the highway. Sometime in 1991, Vivencio Sarmiento, his
daughter Bessie Sarmiento and her husband Beth Del Mundo had a building constructed on a
portion of said government land. In November that same year, a part thereof was occupied by
Andok's Litson Corporation and Marites' Carinderia. In 1993, by means of a Deed of Exchange of
Real Property, Villarico acquired a 74.30 square meter portion of the same area owned by the
government. The property was registered in his name as T.C.T. No. 74430 in the Registry of
Deeds of Parañaque City. In 1995, Villarico filed with the RTC a complaint for accion publiciana
against respondents. He alleged inter alia that respondents' structures on the government land
closed his "right of way" to the Ninoy Aquino Avenue; and encroached on a portion of his lot
covered by T.C.T. No. 74430.

Issue: Whether or not Villarico has a right of way to the NAA.

Ruling: It is not disputed that the lot on which petitioner's alleged "right of way" exists belongs
to the state or property of public dominion. Property of public dominion is defined by Article
420 of the Civil Code as follows: "ART. 420. The following things are property of public
dominion: (1) Those intended for public use such as roads, canals, rivers, torrents, ports and
bridges constructed by the State, banks, shores, roadsteads, and other of similar character. (2)
Those which belong to the State, without being for public use, and are intended for some public
service or for the development of the national wealth." Public use is "use that is not confined to
privileged individuals, but is open to the indefinite public."

Records show that the lot on which the stairways were built is for the use of the people as
passageway to the highway. Consequently, it is a property of public dominion. Property of
public dominion is outside the commerce of man and hence it: (1) cannot be alienated or leased
or otherwise be the subject matter of contracts; (2) cannot be acquired by prescription against
the State; (3) is not subject to attachment and execution; and (4) cannot be burdened by any
voluntary easement. Considering that the lot on which the stairways were constructed is a
property of public dominion, it cannot be burdened by a voluntary easement of right of way in
favor of Villarico. In fact, its use by the public is by mere tolerance of the government through
the DPWH. Villarico cannot appropriate it for himself. Verily, he cannot claim any right of
possession over it.
Characteristics Of Properties Of Public Dominion

MENCHAVEZ VS. TEVES, JR


449 SCRA 380
January 26, 2005
Facts: Menchavez and Teves entered into a Contract of Lease for an area covered for a fishpond
application for a period of five years. During this period, Cebu RTC sheriffsdemolished the
fishpond dikes constructed by Teves. As a consequence, Teves filed for damages with
application for preliminary attachment against Menchavez. In hisComplaint, he alleged that the
lessors had violated their Contract of Lease, specificallythe provision on peaceful and adequate
enjoyment of the property for the entire durationof the Contract. Respondent further asserted
that the lessors had withheld from him thefindings of the trial court in Civil Case No. 510-T,
entitled "Eufracia Colongan andPaulino Pamplona v. Juan Menchavez Sr. and Sevillana S.
Menchavez." In that caseinvolving the same property, subject of the lease, the Menchavez
spouses were orderedto remove the dikes illegally constructed and to pay damages and
attorney's fees.

Issue: Whether or not Menchavez is liable for Teves for the sheriff’s act of demolishing the
constructed dikes.

Ruling: No. A void contract is deemed legally non-existent. It produces no legal effect. As a
general rule, courts leave parties to such a contract as they are, because they arein pari delicto
or equally at fault. Neither party is entitled to legal protection.
DACANAY vs. ASISTIO, JR.
G.R. No. 93654
May 6, 1992

Facts: This is a petition for mandamus to the non-action of the city government of Caloocan in
accordance with the decision of the RTC to evict the occupants of a flea market located in the
streets of Caloocan.

January 5, 1979 – Metropolitan Manila Commission enacted an ordinance allowing the use of
streets for the purpose of flea markets subject to several conditions.

1987 – Mayor Martinez caused the demolition of the flea markets and the stallowners filed a
case against such action.

RTC dismissed the case on the ground that the streets in questions (Heros del '96, Gozon and
Gonzales) are of public dominion, hence outside the commerce of man.

After the decision came out, there was a change in the city administration and current mayor
(Asistio) did not pursue the action of the previous mayor and left the flea markets in the streets
as is.

Dacanay, being a resident of Heroes del '96 filed a petition for mandamus to remove the stalls
in their street

Issue: May public streets be leased or licensed to market stallholders by virtue of a city
ordinance or resolution of Metropolitan Manila Commission?

Held: NO

1. A public street is property for public use hence outside the commerce of man. Being outside
the commerce of man, it may not be the subject of lease or other contract
2. The vested right of the public to use city streets for the purpose they were intended to serve
such as for traveling
3. Any executive order or city resolution cannot change the nature of the public street because
it is going to be contrary to the general law
MANILA LODGE NO. 761 VS. CA
G.R. No. L-41001
September 30, 1976

Facts: The Philippine Commission enacted Act No. 1360 which authorized the City of Manila to
reclaim a portion of Manila Bay. Subsequently Act No. 1657 amended the former act which
states that the City of Manila was authorized to sell or lease the set aside for hotel site. The City
of Manila sells the land to Manila Lodge No. 761 then the latter sold the land to Tarlac
Development Corporation. The City of Manila filed a petition for re-annotation of its right to
repurchased. The TDC then filed a complaint that the City of Manila was estopped from
repurchasing the property.

Issue: Whether or not the City of Manila was estopped from questioning the validity of the sale.

Ruling: The Government is never estopped by mistakes or errors on the pan of its agents, and
estoppel does not apply to a municipal corporation to validate a contract that is prohibited by
law or is against Republic policy, and the sale executed by the City of Manila to Manila Lodge
was certainly a contract prohibited by law. Moreover, estoppel cannot be urged even if the City
of Manila accepted the benefits of such contract of sale and the Manila Lodge No. 761 had
performed its part of the agreement, for to apply the doctrine of estoppel against the City of
Manila in this case would be tantamount to enabling it to do indirectly what it could not do
directly.

The sale of the subject property executed by the City of Manila to the Manila Lodge No. 761,
BPOE, was void and inexistent for lack of subject matter. It suffered from an incurable defect
that could not be ratified either by lapse of time or by express ratification. The Manila Lodge
No. 761 therefore acquired no right by virtue of the said sale. Hence to consider now the
contract inexistent as it always has seen, cannot be, as claimed by the Manila Lodge No. 761, an
impairment of the obligations of contracts, for there was it, contemplation of law, no contract
at all.
REPUBLIC VS. CA
G.R. No. L-61647
October 12, 1984

Facts: The respondents (Tancinco’s) were registered owners of a parcel of land in Bulacan,
bordering on the Maycauayan and Bocaue Rivers. They filed an application for the registration
of three lots adjacent to their fishpond, but because of the recommendation of the
Commissioner, they only pushed for the registration of two. The RTC and CA granted the
petition despite the opposition of the Bureau of Lands.

The respondents based their claim on accretions to their fishponds. They presented a lone
witness (their overseer). The Bureau of Lands argue that the lands in dispute are not accretions.
They assert that what actually happened was that the respondents simply transferred their
dikes simply further down the river bed of the Meycauayan River. Thus, if there was any
accretion to speak of, it was man-made.

Respondents counter that the their evidence shows that accretion happened without human
intervention and that the transfer of the dikes occurred only after.

Issue: Whether or not accretion took place.

Ruling: No. Alluvion must be the exclusive work of nature. There is not evidence that the
addition to said property was made gradually through the effects of the currents of the two
rivers. The lands in question total almost 4 hectares of land, which are highly doubtful to have
been caused by accretion. The lone witness testified that she observed an increase in the area
in 1939, but the lots in question were not included in the survey of their adjacent property
conducted in 1940. They were also not included in the Cadastral Survey of the entire
Municipality of Maycauayan between the years 1958-1960. If the overseer was indeed telling
the truth, the accretion was sudden, not gradual. When the respondents transferred their dikes
towards the river beds, the dikes were meant for reclamation purposes and not to protect their
property from the destructive force of the waters of the river. The lots in question were
portions of the bed of the Meycauayan River and are therefore classified as public property.
Manila International Airport Authority vs. Court of Appeals
G.R. No. 15560
July 20, 3006

Facts: MIAA received Final Notices of Real Estate Tax Delinquency from the City of Parañaque
for the taxable years 1992 to 2001. MIAA’s real estate tax delinquency was estimated at P624
million. The City of Parañaque, through its City Treasurer, issued notices of levy and warrants of
levy on the Airport Lands and Buildings. The Mayor of the City of Parañaque threatened to sell
at public auction the Airport Lands and Buildings should MIAA fail to pay the real estate tax
delinquency.

MIAA filed a petition sought to restrain the City of Parañaque from imposing real estate tax on,
levying against, and auctioning for public sale the Airport Lands and Buildings.

The City of Parañaque contended that Section 193 of the Local Government Code expressly
withdrew the tax exemption privileges of “government-owned and-controlled corporations”
upon the effectivity of the Local Government Code. Thus, MIAA cannot claim that the Airport
Lands and Buildings are exempt from real estate tax.

MIAA argued that Airport Lands and Buildings are owned by the Republic. The government
cannot tax itself. The reason for tax exemption of public property is that its taxation would not
inure to any public advantage, since in such a case the tax debtor is also the tax creditor.

Issue: Whether or not the City of Parañaque can impose real tax, levy against and auction for
public sale the Airport Lands and Buildings.

Ruling: MIAA is Not a Government-Owned or Controlled Corporation. The Airport Lands and
Buildings of MIAA are property of public dominion and therefore owned by the State or the
Republic of the Philippines. No one can dispute that properties of public dominion mentioned
in Article 420 of the Civil Code, like “roads, canals, rivers, torrents, ports and bridges
constructed by the State,” are owned by the State. The term “ports” includes seaports and
airports. The MIAA Airport Lands and Buildings constitute a “port” constructed by the State.

Under Article 420 of the Civil Code, the MIAA Airport Lands and Buildings are properties of
public dominion and thus owned by the State or the Republic of the Philippines. The Airport
Lands and Buildings are devoted to public use because they are used by the public for
international and domestic travel and transportation. The fact that the MIAA collects terminal
fees and other charges from the public does not remove the character of the Airport Lands and
Buildings as properties for public use. The charging of fees to the public does not determine the
character of the property whether it is of public dominion or not. Article 420 of the Civil Code
defines property of public dominion as one “intended for public use.”

VERGO D. TUFEXIS vs. FRANCISCO


G.R. No. L-9865
December 24, 1915

Facts: In the complaint filed by counsel for Vergo D. Tufexis, it was alleged that on September
30, 1911, plaintiff acquired at a public sale held in execution of a judgment rendered against
Ricardo Pardo y Pujol, a piece of property situated in the municipality of Guinobatan, consisting
of a frame building of strong materials with a galvanized-iron roof, erected on a parcel of land
belonging to that municipality and intended for a public market; that plaintiff also acquired at
the sale all the right, interest, title, and participation in the said property that appertained or
might appertain to Pardo Pujol; that the said building was constructed by virtue of a concession
granted by the former Spanish government to Ricardo Pardo y Cabañas, father of the judgment
debtor, who, by a public instrument ofJuly 31, 1912, renounced his right to redeem the said
property and conveyed it to plaintiff, together with all his rights therein, the instrument of
grant, Exhibit A, being attached to the complaint as a part thereof.

Issue: Whether a the subject building on land belonging to the municipality of Guinobatan
which was intended for a public market, by virtue of a concession could be attached and sold
for the payment of a certain debt owed by Ricardo Pardo y Pujol to a third person who had
obtained a final judgment.

Ruling: No, the creditor cannot attach the debt owned by the government.
The supreme court of Alabama, in deciding a similar case (Gardner vs.Mobile & Northwestern
R.R. Co., 102 Ala., 635, 645), affirmed the same principle and said: The only remedy of a
judgment creditor is to obtain the appointment of a receiver and the sequestration of its
income or earnings. It is to be noted that section 56 of Act No. 1459, which permits the sale
under execution of a corporation's franchise, is in no wise applicable to the case at bar, for the
reason that, since this Act was promulgated on March 1, 1906, it could not and cannot affect
the laws, decrees, and orders of the Spanish government in conformity with which the
administrative concession, Exhibit A, was granted to Pardo y Cabañas.

The operation of a railroad is of public interest, and concerns both the public and the state,
even though the superintendent and management thereof be conducted by a private company.
Therefore, the property of a railroad, either its rolling stock or permanent equipment, is not
subject to attachment and sale, and the rights of the creditors of the operating company may
be exercised for the collection of their credit only of the gross receipts after the operation of
the railroad is insured from its own income.
Kinds of Properties of Public Dominion

ROMAN R. SANTOS vs.HON. FLORENCIO MORENO


G.R. No. L-15829
December 4, 1967

Facts: The Zobel family of Spain formerly owned vast track of marshland in the municipality of
Macabebe, Pampanga province. Called Hacienda San Esteban, it was administered and
managed by the Ayala y Cia. From the year 1860 to about the year 1924 Ayala y Cia., devoted
the hacienda to the planting and cultivation of nipa palms from which it gathered nipa sap or
"tuba." It operated a distillery plant in barrio San Esteban to turn nipa tuba into potable alcohol,
which was in turn manufactured into liquor.

Accessibility through the nipa palms deep into the hacienda posed as a problem. Ayala y Cia.,
therefore dug canals leading towards the hacienda's interior where most of them interlinked
with each other. The canals facilitated the gathering of tuba and the guarding and patrolling of
the hacienda by security guards called "arundines." By the gradual process of erosion these
canals acquired the characteristics and dimensions of rivers.
In 1924 Ayala y Cia shifted from the business of alcohol production to bangus culture. It
converted Hacienda San Esteban from a forest of nipa groves to a web of fishponds. To do so, it
cut down the nipa palm, constructed dikes and closed the canals criss-crossing the hacienda.
Sometime in 1925 or 1926 Ayala y Cia., sold a portion of Hacienda San Esteban to Roman
Santos who also transformed the swamp land into a fishpond. In so doing, he closed and built
dikes across Sapang Malauling Maragul, Quiñorang Silab, Pepangebunan, Bulacus, Nigui and
Nasi.

The closing of the man-made canals in Hacienda San Esteban drew complaints from residents of
the surrounding communities. Claiming that the closing of the canals caused floods during the
rainy season, and that it deprived them of their means of transportation and fishing grounds,
said residents demanded re-opening of those canals.

Subsequently, Mayor Lazaro Yambao of Macabebe, accompanied by policemen and some


residents went to Hacienda San Esteban and opened the closure dikes at Sapang Malauling
Maragul Nigui and Quiñorang Silab.

Issue: Whether or not the streams involved in this case belong to the public domain or to the
owner of Hacienda San Esteban according to law and the evidence submitted to the
Department of Public Works and Communications?

Ruling: A private person may take possession of a watercourse if he constructed the same
within his property.

One and all, the evidence, oral and documentary, presented by Roman Santos in the
administrative proceedings supports the conclusion of the lower court that the streams
involved in this case were originally man-made canals constructed by the former owners of
Hacienda San Esteban and that said streams were not held open for public use. This same
conclusion was reached 27 years earlier by an investigator of the Bureau of Public Works whose
report and recommendations were approved by the Director of Public Works and submitted to
the Secretary of Commerce and Communications.

The streams in question were artificially made, hence of private ownership.


Pursuant to Article 71 of the Spanish Law of Waters of August 3, 1866, and Article 408(5) of the
Spanish Civil Code, channels of creeks and brooks belong to the owners of estates over which
they flow. The channels, therefore, of the streams in question, which may be classified creeks,
belong to the owners of Hacienda San Esteban.

With the exception of Sapang Cansusu, being a natural stream and a continuation of the
Cansusu River, admittedly a public stream, belongs to the public domain. Its closure therefore
by the predecessors of Roman Santos was illegal.

All the other streams, being artificial and devoted exclusively for the use of the hacienda owner
and his personnel, are declared of private ownership. Hence, the dams across them should not
he ordered demolished as public nuisances.
Almagro vs. Kwan
G.R. No. 175806
October 20, 2010

Facts: This case involves a 17,181 square meter land which is known as Lot No. 6278-M located
at Maslog, Sibulan, Negros Oriental and inherited by respondents from their parents who died
in 1976 and 1986respectively. On September 18, 1996, respondents filed with the MTC an
action for recovery of possession and damages against petitioners as well as those who are
occupants within the boundary of the land in controversy. During the pre-trial, the Court and
the parties designated geodetic engineer Suasin to perform the task of verification and
relocation survey of said lot and which was accordingly executed on September 12-13, 2000. In
the written report of Engr. Suasin, it contained, inter alia, that the big portion of the lot is
submerged under the sea and a small portion remain as dry land, and that some of the
defendants have constructed buildings or houses inside the dry land.

In the Court’s judgment dated May 11, 2001, the MTC dismissed the complaint on the ground
that the remaining dry portion of Lot No. 6278-M has become foreshore land and should be
returned to the public domain. Respondents appealed to the RTC, which subsequently
concluded upon conducting two ocular inspections, that the disputed remaining portion is not
foreshore land because it remained dry even during high tides. Petitioners moved for
reconsideration, but were denied. They also filed separate petitions for review with the Court
of Appeals.

Issue: Whether or not the disputed portion of Lot No. 6278-M is still private land or has become
foreshore land which forms part of the public domain.

Ruling: The SC held that the disputed portion is private land and is rightfully owned by
respondents, in contravention to the claim of petitioners that is has become foreshore land
and therefore owned by the public domain, and that they have foreshore lease permits from
the DENR on claimed foreshore land, albeit failing to present evidence to prove validity of such
claim.
Binalay vs. Manalo
195 SCRA 374
March 18, 1991

Facts: Manalo acquired 2 lots which were originally owned by Judge Taccad from 2 different
people. The lot was beside the Cagayan River, which, due to flooding, would place a portion of
the land underwater during the rainy season. On sunny days, however, the land would be dried
up for the entire dry season. When a survey of the land was conducted on a rainy month, a
portion of the land that Manalo bought was then underwater and was thus left unsurveyed and
excluded from Lot 307.

The big picture is this: Cagayan River running from south to north, forks at a certain point to
form two braches and then unites at the other end, further north, to form a narrower strip of
land. The eastern branch of the river cuts through Lot 307, and is flooded during the rainy
season. The unsurveyed portion, on the other hand, is the bed of the eastern branch. The fork
exists only during the rainy season while the “island”/elongated strip of land formed in the
middle of the forks becomes dry and perfect for cultivation when the Cagayan river is at its
ordinary depth. The strip of land in the middle of the fork was labeled Lot 821-822. Lot 821 is
directly opposite Lot 307 and is separated by the eastern branch of the river’s fork.

Manalo claims that Lot 821 belongs to him by way of accretion to the submerged portion of the
land to which it is adjacent. Petitioners (Binalay, et al) who possess the Lot 821, on the other
hand, insist that they own it. They occupy the other edges of the lot along the river bank (i.e.
the fertile portions on which they plant tobacco and other agricultural products) and also
cultivate the western strip during the summer.

Issue: Whether or not Manalo owns Lot 821 by way of accretion.

Ruling: No. The disputed property is not an accretion. It is the action of the heavy rains that
cause the highest ordinary level of waters of the Cagayan River during the rainy season. The
depressed portion is a river bed and is thus considered property of public domain.

The requisites of accretion in article 457 were not satisfied. These are: 1) that the deposition of
the soil or sediment be gradual and imperceptible; 2) that it be the result of the action of the
waters of the river (or sea); and 3) the land where the accretion takes place is adjacent to the
banks of the rivers (or the sea coast). The accretion should’ve been attached to Lot 307 for
Manalo to acquire its ownership. BUT, the claimed accretion lies on the bank of the river; not
adjacent to Lot 307 but directly opposite it – across the river. Aside from that, the dike-like
slopes which were very steep may only be formed by a sudden and forceful action like flooding.
The steep slopes could not have been formed by the river in a slow and gradual manner.
JOSE V. HILARIO, JR., vs. THE CITY OF MANILA
G.R. No. L-19570
April 27, 1967

Facts: Dr. Jose Hilario was the registered owner of a large tract of land— located at Barrio
Guinayang, in San Mateo, Rizal. Upon his death, this property was inherited by his son, herein
plaintiff-appellant Jose Hilario, Jr., to whom a new certificate of title was issued.

During the lifetime of plaintiff's father, the Hilario estate was bounded on the western side by
the San Mateo River. To prevent its entry into the land, a bamboo and lumber post dike or ditch
was constructed on the northwestern side. This was further fortified by a stonewall built on the
northern side. For years, these safeguards served their purpose. However, in 1937, a great and
extraordinary flood occurred which inundated the entire place including the neighboring
barrios and municipalities. The river destroyed the dike on the northwest, left its original bed
and meandered into the Hilario estate, segregating from the rest thereof a lenticular place of
land. The disputed area is on the eastern side of this lenticular strip which now stands between
the old riverbed site and the new course.

On October 22, 1949, plaintiff filed his complaint for injunction and damages against the
defendants City Engineer of Manila, District Engineer of Rizal, the Director of Public Works, and
Engr. Busuego, the Engineer-in-charge of the plant. It was prayed that the latter be restrained
from excavating, bulldozing and extracting gravel, sand and soil from his property and that they
solidarily pay to him P5,000.00 as damages. Defendants' answer alleged, in affirmative defense,
that the extractions were made from the riverbed while counterclaiming with a prayer for
injunction against plaintiff—who, it was claimed, was preventing them from their operations.

Issue: Whether or not the riverbanks considered as public dominion.

Ruling: The SC ruled that not all riverbanks are considered as a public dominion.

Art. 372 of the old Civil Code which provides that — Whenever a navigable or floatable river
changes its course from natural causes and opens a new bed through a private estate, the new
bed shall be of public ownership, but the owner of the estate shall recover it in the event that
the waters leave it dry again either naturally or as the result of any work legally authorized for
this purpose.
MANILA INTERNATIONAL AIRPORT AUTHORITY vs. Court of Appeals
GR NO. 155650
July 20, 2006

Facts: Manila International Airport Authority (MIAA) is the operator of the Ninoy International
Airport located at Paranaque City. The Officers of Paranaque City sent notices to MIAA due to
real estate tax delinquency. MIAA then settled some of the amount. When MIAA failed to settle
the entire amount, the officers of Paranaque city threatened to levy and subject to auction the
land and buildings of MIAA, which they did. MIAA sought for a Temporary Restraining Order
from the CA but failed to do so within the 60 days reglementary period, so the petition was
dismissed. MIAA then sought for the TRO with the Supreme Court a day before the public
auction, MIAA was granted with the TRO but unfortunately the TRO was received by the
Paranaque City officers 3 hours after the public auction.

MIAA claims that although the charter provides that the title of the land and building are with
MIAA still the ownership is with the Republic of the Philippines. MIAA also contends that it is an
instrumentality of the government and as such exempted from real estate tax. That the land
and buildings of MIAA are of public dominion therefore cannot be subjected to levy and auction
sale. On the other hand, the officers of Paranaque City claim that MIAA is a government owned
and controlled corporation therefore not exempted to real estate tax.

Issue: Whether the Airport Lands and Buildings of MIAA are exempt from real estate tax under
existing laws.

Ruling: Under the Local government code, government owned and controlled corporations are
not exempted from real estate tax. MIAA is not a government owned and controlled
corporation, for to become one MIAA should either be a stock or non stock corporation. MIAA
is not a stock corporation for its capital is not divided into shares. It is not a non stock
corporation since it has no members. MIAA is an instrumentality of the government vested with
corporate powers and government functions.

Under the civil code, property may either be under public dominion or private ownership.
Those under public dominion are owned by the State and are utilized for public use, public
service and for the development of national wealth. The ports included in the public dominion
pertain either to seaports or airports. When properties under public dominion cease to be for
public use and service, they form part of the patrimonial property of the State.

The court held that the land and buildings of MIAA are part of the public dominion. Since the
airport is devoted for public use, for the domestic and international travel and transportation.
Even if MIAA charge fees, this is for support of its operation and for regulation and does not
change the character of the land and buildings of MIAA as part of the public dominion. As part
of the public dominion the land and buildings of MIAA are outside the commerce of man. To
subject them to levy and public auction is contrary to public policy. Unless the President issues
a proclamation withdrawing the airport land and buildings from public use, these properties
remain to be of public dominion and are inalienable. As long as the land and buildings are for
public use the ownership is with the Republic of the Philippines.
Reclaimed Properties

G.R. No. 191109 July 18, 2012


REPUBLIC OF THE PHILIPPINES vs. CITY OF PARANAQUE

Facts: In view of the finding of this court that petitioner is not exempt from payment of real
property taxes, respondent Parañaque City Treasurer Liberato M. Carabeo did not act xxx
without or in excess of jurisdiction, or with grave abuse of discretion amounting to lack or in
excess of jurisdiction in issuing the warrants of levy on the subject properties.
The Public Estates Authority (PEA) is a government corporation created by virtue of Presidential
Decree (P.D.) No. 1084 (Creating the Public Estates Authority, Defining its Powers and
Functions, Providing Funds Therefor and For Other Purposes) which took effect on February
4,1977 to provide a coordinated, economical and efficient reclamation of lands, and the
administration and operation of lands belonging to, managed and/or operated by, the
government with the object of maximizing their utilization and hastening their development
consistent with public interest.

On October 26, 2004, then President Gloria Macapagal-Arroyo issued E.O. No. 380 transforming
PEA into PRA, which shall perform all the powers and functions of the PEA relating to
reclamation activities. By virtue of its mandate, PRA reclaimed several portions of the foreshore
and offshore areas of Manila Bay, including those located in Parañaque City, and was issued
Original Certificates of Title (OCT Nos. 180, 202, 206, 207, 289, 557, and 559) and Transfer
Certificates of Title (TCT Nos. 104628, 7312, 7309, 7311, 9685, and 9686) over the reclaimed
lands.

On February 19, 2003, then Parañaque City Treasurer Liberato M. Carabeo (Carabeo) issued
Warrants of Levy on PRA’s reclaimed properties (Central Business Park and Barangay San
Dionisio) located in Parañaque City based on the assessment for delinquent real property taxes
made by then Parañaque City Assessor Soledad Medina Cue for tax years 2001 and 2002. March
26, 2003, PRA filed a petition for prohibition with prayer for temporary restraining order (TRO)
and/or writ of preliminary injunction against Carabeo before the RTC.

Issue: Whether or not the petitioner is exempt from payment of real property tax under.

Ruling: Yes, the petitioner is an incorporated instrumentality of the government however it is


not a GOCC. The facts, the evidence on record and jurisprudence on the issue support the
position that PRA was not organized either as a stock or a non-stock corporation. Neither was it
created by Congress to operate commercially and compete in the private market. Instead, PRA
is a government instrumentality vested with corporate powers and performing an essential
public service pursuant to Section 2(10) of the Introductory Provisions of the Administrative
Code. Being an incorporated government instrumentality, it is exempt from payment of real
property tax.
PATRIMONIAL PROPERTY OF THE STATE

Conversion of Property of Public Dominion to Patrimonial Property

Yu Chang vs. Republic of the Philippines


GR No. 171726
February 23, 2011

Facts: Petitioner’s father L. Yu Chang executed an Agreement to Exchange Real Property with
the Mayor of Pili, Camarines Sur, Justo Casuncad wherein the former assigned and transferred
to the Municipality of Pili his 400-square-meter residential lot in Barrio San Roque, Pili,
Camarines Sur, in exchange for a 400-square-meter piece of land located in San Juan, Pili. Office
of Solicitor General (OSG) filed an opposition, saying that the land is legally classified as Public
Domain, forest land.

Issue: Whether or not the two lots are considered a private land.

Ruling: Section 48 (b) of the Public Land Act as amended by PD 1073 provised that: The
following described citizens of the Philippines, occupying lands of public domain or claiming to
own any such lands or an interest therein, but whose title have not been perfected or
completed, may apply to Regional Trial Court of the province or city where the land is located
for confirmation of their claims and the issuance of a Certificate of title thereof, under the
Property Registration Decree.

In order that petitioner’s application for registration of title may be granted, they must first
establish the following:
1. That the subject land forms part of the disposable and alienable lands of the Public Domain
2. That they have been in open, continuous, exclusive and notorious possession and occupation
of the same under a bona fide claim of ownership, since June 12, 1945.

Jose Amunategui vs Director of Forestry provides that: A forested area classified as forest land
of the public domain does not lose such classification simply because loggers or settlers may
have stripped it of its forest cover. The classification of land is descriptive of its legal nature or
status and does not have to be descriptive of what the land actually looks like.
Laurel v. Garcia
187 SCRA 797
July 25, 1990

Facts: Pres. Corazon Aquino wanted to sell the property located in Roppongi, Japan. She
conducted a bidding for the sale of the property. The property in Roppongi was a gift of
Japanese to the Filipinos who suffered, died, harassed under the hands of the Japanese during
World War ll, then it was accepted by the Filipinos. The sale of the said property was opposed
by the Vice-pres. Laurel. He alleged that the property was a public dominion it could not be sold
because it is belong to the Republic of the Philippines. Pres. Aquino stated that the funds for
the selling of the property would be used for the national economic development projects and
to the CARP projects. So, the vice pres. Laurel filed a petition for annulment of sale of property
located in Roppongi, Japan on the ground that it is a public dominion. Pres. Aquino cited that
the Roppongi Property had been abandoned for a long time over 14 yrs. Ago and it was not yet
renovated.

Issue: Whether or not the property located in Roppongi, Japan was considered a patrimonial
property or public dominion.

Ruling: The SC ruled that the said property was considered as a public dominion. Under the civil
code a property belonged to the state is not used anymore as public service or public purposes
forms part of the patrimonial property. This provision the property will not automatically
change its classification from public dominion to patrimonial property without enacted by the
congress, even the public dominion is abandoned still it must have an enaction by the congress.
Hence the said property located in Roppongi, Japan will not be sold and it would remain
lassified as public dominion.
Accion Reinvindicatoria; Ejectment
Del Fierro v. Seguiran
G.R. NO. 152141
Aug. 8, 2011

Facts:
The complaint alleged that the petitioners were the owners and possessors of a parcel of land
identified as lot 1625 and 1626 formerly part of lot no. 1197 situated at brgy. locloc Zambales.
On july 26, 1964, lodelfo and marcial unlawfully entered the land occupied by the plaintiffs. The
plaintiff sued them for forible entry before the MTC of palauig. The MTC rendered a decision
that in favor of the plaintiff hat the illegal settlers should vacate the premises. The illegal
settlers mortgaged the property to the bank and the other lot sold it to Rene S. Seguiran.

Issue: Whether the petitioners are entitled to reconveyance.

Ruling: No, the requisite of conveyance are provided for in art. 434 of the civil code thus, in an
action to recover, the property must be identified and the plaintiff must rely on the strength of
his tile and not on the weakness of the defendants claim.

Art. 434 of the civil code provides that to successfully maintain an action to recover the
ownership of a real property, the person who claims a better right to must prove two things: 1.
The identity of the land claimed 2. The title thereto. In regard to the first requisite, in an action
reindivicatoria
Del Rosario v. Roxas Foundation, Inc.
G.R. No. 170575
June 8, 2011

Facts: Manuel del Rosario appears to be the registered owner of Lot 3-A of Psd-301974 located
inRoxas City which is described in and covered by a Transfer Certificate of Title (TCT).Sometime
in 1991, the Gerry Roxas Foundation, Inc., (GRFI) as a legitimate foundation, took possession
and occupancy of said land by virtue of a memorandum of agreement it entered with Roxas
City. Its possession and occupancy of said land is in the character of being a lessee thereof.

In February and March 2003, the Spouses Manuel and Florentina del Rosario (Spouses) served
notices upon the GRFI to vacate the premises of said land. GRFI, however, did not heed such
notices because it still has the legal right to continue its possession and occupancy of said land.
On July 2003, the spouses filed a Complaint for Unlawful Detainer against GRFI before the
Municipal Trial Court in Cities. Said complaint contains, among others, the following significant
allegations:

3. Plaintiffs are the true, absolute and registered owners of a parcel of land, situated at
Dayao, Roxas City and covered by and described in TCT No. 18397 issued to the plaintiffs
by the Register of Deeds for Roxas City.
4. Sometime in 1991, without the consent and authority of the plaintiffs, defendant took
full control and possession of the subject property, developed the same and used it for
commercial purposes.
7. Plaintiffs have allowed the defendant for several years, to make use of the land
without any contractual or legal basis. Hence, defendant’s possession of the subject
property is only by tolerance.
8. But plaintiffs’ patience has come to its limits. Hence, sometime in the last quarter of
2002, plaintiffs made several demands upon said defendant to settle and/or pay rentals
for the use of the property.
10. Notwithstanding receipt of the demand letters, defendant failed and refused, as it
continues to fail and refuse to pay reasonable monthly rentals for the use and
occupancy of the land, and to vacate the subject premises despite the lapse of the
fifteen-day period specified in the said demand letters. Consequently, defendant is
unlawfully withholding possession of the subject property from the plaintiffs, who are
the owners thereof.

Issue: Whether or not the allegations in the Complaint establish a cause of action for forcible
entry and not an unlawful detainer?

Ruling: Yes. In forcible entry, one is deprived of physical possession of any land or building by
means of force, intimidation, threat, strategy, or stealth. Where the defendant’s possession of
the property is illegal ab initio the summary action for forcible entry (detentacion) is the
remedy to recover possession.

In their Complaint, the spouses maintained that the GRFI took possession and control of the
subject property without any contractual or legal basis. Assuming that these allegations are
true, it hence follows that GRFI’s possession was illegal from the very beginning. Therefore, the
foundation of the spouses Complaint is one for forcible entry– that is, the forcible exclusion of
the original possessor by a person who has entered without right. Thus, there can be no
tolerance as the spouses alleged that GRFI’s possession was illegal at the inception.

Corollarily, since the deprivation of physical possession, as alleged in the spouses’ Complaint
was attended by strategy and force, this Court finds that the proper remedy for the spouses
was to file a Complaint for Forcible Entry and not the instant suit for unlawful detainer.
FIORELLO R. JOSE vs. ROBERTO ALFUERTO
G.R. No. 169380
November 26, 2012

Facts: The dispute involves a parcel of land registered in the name of Rodolfo Chua Sing under
Transfer Certificate of Title No. 52594, with an area of 1919 square meters, located in Barangay
San Dionisio, Paraque City. Chua Sing purchased the land in 1991. On April 1, 1999, Chua Sing
leased the property to the petitioner. Their contract of lease was neither notarized nor
registered with the Paraque City Registry of Deeds.

The lease contract provided that:


That the term of this lease shall be FIVE (5) years and renewable for the same period upon
mutual agreement of the parties to commence upon the total eviction of any occupant or
occupants. The LESSOR hereby transfers all its rights and prerogative to evict said occupants in
favor of the LESSEE which shall be responsible for all expenses that may be incurred without
reimbursement from the LESSOR. It is understood however that the LESSOR is hereby waiving,
in favor of the LESSEE any and all damages that may be recovered from the occupants.
Significantly, the respondents already occupied the property even before the lease contract
was executed.

On April 28, 1999, soon after Chua Sing and the petitioner signed the lease contract, the
petitioner demanded in writing that the respondents vacate the property within 30 days and
that they pay a monthly rental of P1,000.00 until they fully vacate the property.

The respondents refused to vacate and to pay rent. On October 20, 1999, the petitioner filed an
ejectment case against the respondents before Branch 77 of the Paraque City MeTC, docketed
as Civil Case No. 11344.

In this complaint, no mention was made of any proceedings before the barangay. Jose then
brought the dispute before the barangay for conciliation. The barangay issued a Certification to
File Action on March 1, 2000. Jose was then able to file an amended complaint, incorporating
the proceedings before the barangay before the summons and copies of the complaint were
served upon the named defendants.

Issue: Whether or not an action for unlawful detainer is the proper remedy.

Ruling: Unlawful detainer is a summary action for the recovery of possession of real property.
This action may be filed by a lessor, vendor, vendee, or other person against whom the
possession of any land or building is unlawfully withheld after the expiration or termination of
the right to hold possession by virtue of any contract, express or implied. In unlawful detainer,
the possession of the defendant was originally legal, as his possession was permitted by the
plaintiff on account of an express or implied contract between them. However, the defendants
possession became illegal when the plaintiff demanded that the defendant vacate the subject
property due to the expiration or termination of the right to possess under the contract, and
the defendant refused to heed such demand. A case for unlawful detainer must be instituted
one year from the unlawful withholding of possession.

It is not the first time that this Court adjudged contradictory statements in a complaint for
unlawful detainer as a basis for dismissal. In Unida v. Heirs of Urban, the claim that the
defendants possession was merely tolerated was contradicted by the complainants allegation
that the entry to the subject property was unlawful from the very beginning. The Court then
ruled that the unlawful detainer action should fail.
Barrientos v.Rapal
G.R. NO. 169594
July 20, 2011

Facts: On April 15, 1998, respondent Mario Rapal acquired a parcel of land at Quezon city via a
notarized Deed of Transfer of Possessory Right. The parcel of land was said to be aportion of
the estate of the late Don Mariano San Pedro y Esteban. Thereafter, respondent constructed a
semi-concrete house on the lot and took actual possession of the property by himself and
through his caretaker, Benjamin Tamayo.

In 1993, Petitioner Bienvinido Barrientos and his family were allowed to stay in the subject
property as caretakers with a condition that they shall vacate the premises when the respond
would need it. Upon demand to vacate, petitioner refused to leave the subject property. Thus,
respondent filed a complaint of Unlawful Detainer against the petitioner. On February 21, 2000,
the trial court rendered a decision in favor of the respondent. It ordered petitioner to vacate
the premises and to pay respondent a compensation for the use of the structure. On appeal,
(RTC) reversed the Decision of the MeTC and resolved in favor of petitioner, reasoning that
respondent has not shown any prior lawful possession of the property in question.

On April 29, 2005, the CA rendered the assailed Decision reversing the decision of the RTC and
reinstating the decision of the MeTC. The CA found that both parties presented weak evidence
of ownership. In determining who between the parties was first in possession, the CA
concluded that respondent was, indeed, first in possession of the lot. Hence, petitioner
elevated the case to the SC.

Issues: whether the issue of ownership can be initially resolved for the purpose of determining
the issue of possession.

Ruling: Both petitioner and respondent presented weak evidence of ownership. Respondent on
his part based his claim of ownership over the subject property on the strength of a notarized
Deed of Transfer of Possessory Right from a certain Antonio Natavio. However, respondent
could not derive any right therefrom as the original certificate of title over the land was
declared null and void. Petitioner, on the other hand, anchored his contention that he has a
better right to possess the property on the fact the he is in actual possession of the property.
Yet he failed to adduce sufficient evidence on the manner by which he acquired ownership.
Based on the evidences presented, it can be deduced that petitioner's occupation of the subject
lot was by mere tolerance only. Petitioner was initially permitted by respondent to occupy the
lot as a care taker. Therefore, he is bound by an implied promise that he will vacate the same
upon demand, failing which a summary action for ejectment is the proper remedy.
Doctrine of Self Help

German Management and Services vs. Court Of Appeals


177 SCRA 495
1989

Facts: Spouses Jose issued a power of attorney in favor of petitioner for the
development of their parcel of land into a subdivision. Private respondents were occupying
the land and petitioner advised them to vacate but they refused. Thereafter, petitioner
continued their development and construction. Respondents then filed a case for
forcible entry. The trial court dismissed the complaint and this was reversed by the CA.

Issue: Whether or not the doctrine of self-help is applicable in this case.

Ruling: Notwithstanding petitioner’s claim that it was duly authorized by the owners to
develop the subject property, private respondents as actual possessors, can commence a
forcible entry case against petitioner because ownership is not in issue. Forcible entry is merely
a quieting process, and never determines the actual title to an estate. Title is not involved.

Although admittedly petitioner may validly claim ownership based on the muniment of
title it presented, such evidence doesn’t responsively address the issue of prior actual
possession raised in a forcible entry case. It must be stated that regardless of the actual
condition of title to the property, the party in a peaceable quiet possession shall not be turned
out by a strong hand, violence or terror. Thus, a party who can prove prior possession can
recover such possession even against the owner himself. Whatever may be the character of his
prior possession, if he has in favor priority in time, he has security that entitles him to remain
on the property until he has been lawfully ejected by a person having a better right by accion
publiciana or accion reivindicatoria.
Right To Sub-Surface and Airspace

Republic of the Philippines vs. Court of Appeals


160 SCRA 228

Facts: Jose dela Rosa filed an application for registration of a parcel of land on his own behalf
and on behalf of his children. This application was separately opposed by Benguet
Consolidated, Inc. (Benguet) and Atok Big Wedge Corporation (Atok).

The petitioners claimed that they have acquired the land from their parents and that they have
been in possession of the land ever since. Benguet and Atok opposed on the ground that they
have mineral claims covering the property and had been in actual, continuous and exclusive
possession of the land in concept of owner.

The trial court denied the application while the Court of Appeals reversed the decision of the
trial court and recognized the claims of the applicant but subject to the rights of Benguet and
Atok respecting their mining claims. In other words, the Court of Appeals affirmed the surface
rights of the de la Rosas over the land while at the same time reserving the sub-surface rights of
Benguet and Atok by virtue of their mining claims.

Issue: Whether or not the CA's ruling was correct.

Ruling: No, the CA was incorrect. Art. 437. The owner of a parcel of land is the owner of its
surface and of everything under it, and he can construct thereon any works or make any
plantations and excavations which he may deem proper, without detriment to servitudes and
subject to special laws and ordinances. He cannot complain of the reasonable requirements of
aerial navigation.

Under the theory of the respondent court, the surface owner will be planting on the land while
the mining locator will be boring tunnels underneath. The farmer cannot dig a well because he
may interfere with the operations below and the miner cannot blast a tunnel lest he destroy
the crops above. How deep can the farmer, and how high can the miner, go without
encroaching on each other's rights? Where is the dividing line between the surface and the sub-
surface rights?

It is a well-known principle that the owner of piece of land has rights not only to its surface but
also to everything underneath and the airspace above it up to a reasonable height.

The rights over the land are indivisible and that the land itself cannot be half agricultural and
half mineral. The classification must be categorical; the land must be either completely mineral
or completely agricultural.

In the instant case, as already observed, the land which was originally classified as forest land
ceased to be so and became mineral — and completely mineral — once the mining claims were
perfected. As long as mining operations were being undertaken thereon, or underneath, it did
not cease to be so and become agricultural, even if only partly so, because it was enclosed with
a fence and was cultivated by those who were unlawfully occupying the surface.
National Power Corporation vs. Ibrahim
526 SCRA 149
June 29, 2007

Facts: Ibrahim owns a parcel of land located in Lanao del Norte. In 1978, NAPOCOR took
possession of the sub-terrain area of the land and constructed underground tunnels on the said
property. The tunnels were apparently being used by NAPOCOR in siphoning the water of Lake
Lanao and in the operation of NAPOCOR’s Agus projects.

In 1991, Maruhom (one of the co-heirs of Ibrahim) requested Marawi City Water District for a
permit to construct or install a motorized deep well on the parcel of land but it was rejected on
the grounds that the construction would cause danger to lives and property by reason of the
presence of the underground tunnels. Maruhom demanded NAPOCOR to pay damages and to
vacate the sub-terrain portion of the land.

Issue: Whether or not Ibrahim is the rightful owner of the sub-terrain area of the land.

Ruling: YES. The sub-terrain portion of the property belongs to Ibrahim.

The Supreme Court cited Article 437 of the Civil Code which provides that: The owner of a
parcel of land is the owner of its surface and of everything under it, and he can construct
thereon any works or make any plantations and excavations which he may deem proper,
without detriment to servitudes and subject to special laws and ordinances. xxx

Hence, the ownership of land extends to the surface as well as to the subsoil under it.
Therefore, Ibrahim owns the property as well as the sub-terrain area of the land where the
underground tunnels were constructed.

On the issue of just compensation, the Supreme Court also said that Ibrahim should be paid a
just compensation.

Ibrahim could have dug upon their property and built motorized deep wells but was prevented
from doing so by the authorities because of the construction of the tunnels underneath the
surface of the land.

Ibrahim still had a legal interest in the sub-terrain portion insofar as they could have excavated
the same for the construction of the deep wells. It has been shown that the underground
tunnels have deprived the plaintiffs of the lawful use of the land and considerably reduced its
value.

It was held that: If the government takes property without expropriation and devotes the
property to public use, after many years, the property owner may demand payment of just
compensation in the event restoration of possession is neither convenient nor feasible. This is
in accordance with the principle that persons shall not be deprived of their property except by
competent authority and for public use and always upon payment of just compensation.
Right to Hidden Treasure

Palero-Tan v. Urdaneta
AM NO. P-‐07-‐2399
Jun. 18, 2008

Facts: Urdaneta found jewelry inside the RTC office. Tan is the owner of the jewelry. Urdaneta
did not return the jewelry instead he threw it away when his wife suspected that we bought
the jewelry for a mistress. However, it was found out that he knew that Tan had lost the jewelry
and that he did not bother to inform her.
Issue: Whether or not Urdaneta can be faulted for throwing away the jewelry he found.

Ruling: Yes, he can be faulted. When a person who finds a thing that has been lost or mislaid
by the owner takes the thing into his hands, he acquires physical custody only and does not
become vested with legal possession. In assuming such custody, the finder is charged with the
obligation of restoring the thing to its owner. It is thus respondent’s duty to report to his
superior or his officemates that he found something. The Civil Code, in Article 719, explicitly
requires the finder of a lost property to report it to the proper authorities, thus: Article 719.
Whoever finds a movable, which is not treasure, must return it to its previous possessor. If the
latter is unknown, the finder shall immediately deposit it with the mayor of the city or
municipality where the finding has taken place. The finding shall be publicly announced by the
mayor for two consecutive weeks in the way he deems best. If the movables cannot be kept
without deterioration, or without the expenses which considerably diminish its value, it shall be
sold at public auction eight days after the publication. Six months from the publication having
elapsed without the owner having appeared, the thing found, or its value, shall be awarded to
the finder. The finder and the owner shall be obliged, as the case may be, to reimburse the
expenses.
Rules Governing BPS

FLOREZA vs. EVANGELISTA


G.R. No. L-25462
February 21, 1980

Facts: Plaintiffs Maria de Evangelista and Sergio Evangelista, who are mother and son, are the
owners of a residential lot with an area of 204.08 sq. ms., assessed at P410.00. In May 1945, the
EVANGELISTAS borrowed from FLOREZA the amount of P100.00. On or about November 1945,
with the consent of the EVANGELISTAS, FLOREZA occupied the above residential lot and built
thereon a house of light materials without any agreement as to payment for the use of said
residential lot owing to the fact that the EVANGELISTAS has then a standing loan of P100.00 in
favor of FLOREZA.

On the following dates, the EVANGELISTAS again borrowed the indicated amounts: September
16, 1946 — P100.00; 2 August 17, 1947 — P200,00; 3 January 30, 1949 — P200.00; 4 April 1,
1949 — P140.00, 5 or a total of P740.00 including the first loan. The last three items are
evidenced by private documents stating that the residential lot stands as security therefor and
that the amounts covered thereunder are payable within six years from date, without mention
of interest. The document executed on September 16, 1946 stated specifically that the loan was
without interest "walang anumang patubo."

On January 10, 1949, FLOREZA demolished this house of light materials and in its place
constructed one of strong materials assessed in his name at P1,410.00 under a Tax Declaration.
FLOREZA paid no rental as before.

On August 1, 1949, the EVANGELISTAS, for and in consideration of P1,000.00 representing the
total outstanding loan of P740.00 plus P260.00 in cash, sold their residential lot to FLOREZA,
with a right to repurchase within a period of 6 years from date, or up to August 1, 1955, as
evidenced by a notarial document, Exh. B, registered under Act 3344 on December 6, 1949, as
Inscription No. 2147. 7

Issue: Whether or not the article 448 of the civil code applicable in this case.

Ruling: No, Art. 448 of the civil code is not applicable in this case. The Supreme Court upholds
the Court of Appeals’ conclusion that Article 448 of the Civil Code is inapplicable to the factual
milieu herein. Said codal provision applies only when the builder, planter, or sower believes he
had the right so to build, plant or sow because he thinks he owns the land or believes himself to
have a claim of title. In this case, petitioner makes no pretensions of ownership whatsoever.

The house was already there at the tolerance of the EVANGELISTAS in consideration of the
several loans extended to them. Since petitioner cannot be classified as a builder in good faith
within the purview of Article 448 of the Civil Code, nor as a vendee a retro, who made useful
improvements during the lifetime of the pacto de retro, petitioner has no right to
reimbursement of the value of the house which he had erected on the residential lot of the
EVANGELISTAS, much less to retention of the premises until he is reimbursed.The rights of
petitioner are more akin to those of a usufructuary who, under Article 579 of the Civil (Art. 487
of the old Code), may make on the property useful improvements but with no right to be
indemnified therefor. He may, however, remove such improvements should it be possible to do
so without damage to the property: For if the improvements made by the usufructuary were
subject to indemnity, we would have a dangerous and unjust situation in which the
usufructuary could dispose of the owner's funds by compelling him to pay for improvements
which perhaps he would not have made.
Mercado vs. CA
G.R. No. L-44001
June 10, 1988

Facts: The private respondents Bulaong Group, had for many years been individual lessees of
stalls in the public market of Baliuag, Bulacan; from 1956 to 1972. The market was destroyed by
fire on February 17, 1956; the members of the Bulaong Group constructed new stalls therein at
their expense; and they thereafter paid rentals thereon to the Municipality of Baliuag.
In 1972, the members of the group sub-leased their individual stalls to other persons, referred
to as the Mercado Group. After the Mercado Group had been in possession of the market stalls
for some months, as sub-lessees of the Bulaong Group, the municipal officials of Baliuag
cancelled the long standing leases of the Bulaong Group and declared the persons comprising
the Mercado Group as the rightful lessees of the stalls in question, in substitution of the former.

The members of the Bulaong Group sued. They filed several individual complaints with the
Court of First Instance seeking recovery of their stalls from the Mercado Group as well as
damages. Their theory was anchored on their claimed ownership of the stalls constructed by
them at their own expense, and their resulting right, as such owners, to sub-lease the stalls, and
necessarily, to recover them from any person withholding possession thereof from them.

Issue: Whether or not Bulaong Group is in good faith.

Ruling: It was held that “to be deemed a builder in good faith, it is essential that a person assert
title to the land on which he builds; i.e., that he be a possessor in concept of owner, and that he
be unaware ‘that there exists in his title or mode of acquisition any flaw which invalidates it.’

The members of the Bulaong group were admittedly lessees of space in the public market; they
therefore could not, and in truth never did make the claim, that they were owners of any part
of the land occupied by the market so that in respect of any new structure put up by them
thereon, they could be deemed builders in good faith (in accordance with Article 526 of the
Civil Code). To be deemed a builder in good faith, it is essential that a person assert title to the
land on which he builds; i.e., that he be a possessor in concept of owner, and that he be
unaware “that there exists in his title or mode of acquisition any flaw which invalidates it. It is
such a builder in good faith who is given the right to retain the thing, even as against the real
owner, until he has been reimbursed in full not only for the necessary expenses but also for
useful expenses. On the other hand, unlike the builder in good faith, a lessee who “makes in
good faith useful improvements which are suitable to the use for which the lease is intended,
without altering the form or substance of the property leased,” can only claim payment of
“one-half of the value of the improvements” or, “should the lessor refuse to reimburse said
amount, remove the improvements, even though the principal thing may suffer damage
thereby.”
BALUCANAG vs FRANCISCO
G.R. No. L-33422
May 30, 1983

FACTS: The petitioner bought a lot owned by Mrs. Charvet which was then previously leased by
the latter to one Richard Stohner. The said lease contract provided that the lessee may erect
structures and improvements which shall remain as lessee's property and he may remove them
at any time.

It further provided that should the lessee fail to remove the same structures or improvements
within two months after the expiration of the lease, the lessor may remove them or cause them
to be removed at the expense of the lessee.

Stohner made fillings on the land and constructed a house. When he failed to pay the rent, the
petitioner, through counsel, sent Stohner a demand letter ordering him to vacate the lot. The
lessee contended that he is a 'builder in good faith.'

ISSUE: Whether or not the lessee a builder in good faith?

RULING: The lessee cannot be considered a builder in good faith. The provision under Art. 448
of the New Civil Code (Philippine) on a builder of good faith applies only to the owner of the
land who believes he is the rightful owner thereof, but not to a lessee who's interest in the land
is derived only from a rental contract. Neither can Stohner be considered a 'possessor in good
faith'.

A possessor in good faith is a party who possesses property believing that he is its rightful
owner but discovers later on a flaw in his title that could indicate that he might not be its legal
owner. It cannot apply to a lessee because he knows right from the start that he is merely a
lessee and not the owner of the premises. As a mere lessee, he introduces improvements to the
property at his own risk such that he cannot recover from the owner the reimbursements nor
he has any right to retain the premises until reimbursements. What applies in this case is Art.
1678 (NCC) which provides that, " if the lessee, makes, in good faith, useful improvements
which are suitable to the use for which the lease is intended, without altering the form or
substance of the property leased, the lessor upon the termination of the lease shall pay the
lessee 1/2 of the value of the improvements at the time. Should the lessor refuse to reimburse
said amount, the lessee may remove the improvements even though the principal thing may
suffer damage thereby. He shall not. However cause any more impairment upon the property
leased than is necessary."
PNB vs. DE JESUS
411 SCRA 557

Facts: On 10 June 1995, respondent filed a complaint against petitioner before the Regional
Trial Court for recovery of ownership and possession, with damages, over a portion of the land
covered by Transfer Certificate of Title (TCT) No. T-17197. In his complaint, respondent stated
that he had acquired a parcel of land with an area of 1,144 square meters covered on 26 March
1993. He had caused a verification survey of the property and discovered that the northern
portion of the lot was being encroached upon by a building of petitioner to the extent of 124
square meters. Despite two letters of demand sent by respondent, petitioner failed and refused
to vacate the area.

Petitioner, in its answer, asserted that when it acquired the lot and the building sometime in
1981 from then Mayor Bienvenido Ignacio, the encroachment already was in existence and to
remedy the situation, Mayor Ignacio offered to sell the area in question (which then also
belonged to Ignacio) to petitioner at P100.00 per square meter which offer the latter claimed to
have accepted. The sale, however, did not materialize when, without the knowledge and
consent of petitioner, Mayor Ignacio later mortgaged the lot to the Development Bank of the
Philippines.

Issue: Whether or not the petitioner is a builder in good faith.

Ruling: Yes, the petitioner is a builder in good faith. A builder in good faith can, under the
foregoing provisions, compel the landowner to make a choice between appropriating the
building by paying the proper indemnity or obliging the builder to pay the price of the land. The
choice belongs to the owner of the land, a rule that accords with the principle of accession, i.e.,
that the accessory follows the principal and not the other way around. Even as the option lies
with the landowner, the grant to him, nevertheless, is preclusive. He much choose one. He
cannot, for instance, compel the owner of the building to instead remove it from the land. In
order, however, that the builder can invoke that accruing benefit and enjoy his corresponding
right to demand that a choice be made by the landowner, he should be able to prove good faith
on his part.

Good faith, here understood, is an intangible and abstract quality with no technical meaning or
statutory definition, and it encompasses, among other things, an honest belief, the absence of
malice and the absence of design to defraud or to seek an unconscionable advantage. An
individual personal good faith is a concept of his own mind and, therefore, may not conclusively
be determined by his protestations alone. It implies honesty of intention, and freedom from
knowledge of circumstances which ought to put the holder upon inquiry. The essence of good
faith lies in an honest belief in the validity of ones right, ignorance of a superior claim, and
absence of intention to overreach another. Applied to possession, one is considered in good
faith if he is not aware that there exists in his title or mode of acquisition any flaw which
invalidates it.
Javier v. Concepcion, Jr
G.R. No. L-36566
November 7, 1979

FACTS: On October 17, 1959, respondents as plaintiffs, Lim Chua, Tan Tian On alias Tan Tian
Una and Tan Sick Tan alias Tan Shiok Tuan filed against herein petitioners, then defendants-
spouses, Urbano Javier and Leonila Albiela, with the Court of First Instance of the Province of
Quezon, Civil Case No. 6253, for the reconveyance to the former of a parcel of land with
improvements thereon, known as Lot 12 consisting of fifty (50) hectares, more or less, and an
accounting and recovery of the produce of the land from the time the latter, i.e., petitioners
herein, took possession of the same in 1945 up to the time possession is returned to the
former.

Defendants-petitioners claimed P5,000.00 for attorney's fees and P1,000.00 for litigation
expenses and that in the event that plaintiffs-respondents are declared the lawful owners of
the lot in question, they be reimbursed the amount of P150,000.00 for the reasonable value of
improvements they introduced thereon consisting of a house, camarin made of strong
materials and various fruit trees.

With respect to the claim of the defendants that they acquired the property by prescription,
the same is without merit taking into consideration that the defendant Urbano Javier knew that
the property in question is within lot No. 6 and covered by a certificate of title in favor of the
plaintiffs since 1924 when he filed an opposition to the registration of the land in question and,
therefore, could not be said to have acted in good faith for the purpose of applying the
provision of the Civil Code in ordinary prescription.

ISSUE: Whether or not in holding that the cause of action of the private respondents has not
been barred by the Statute of Limitation or by laches; and

RULING: It can be readily seen that in the above-cited case the land in question came into the
possession of the defendant-appellant Gamponia after a series of transfers from Domingo
Mejia, the original owner and plaintiff-appellee's (Mejia de Lucas') predecessor-in-interest to
three other persons and their successors-in-interest, whose rights and obligation would have
been affected by a contrary decision. Here, there are no intervening rights of third persons
which may be affected or prejudiced by a decision directing the return of Lot No. 12 to
plaintiffs-respondents, hence the equitable defense of laches will not also apply as against the
registered owners in this case.
Nuguid vs. Court of Appeals
452 SCRA 243, 252
2005

FACTS: The deceased spouses Victorino and Crisanta dela Rosa (spouses dela Rosa) were
registered owners of a parcel of land in Orani, Bataan, and covered by OCT No. 3778. On or
about May 4, 1931, Victorino dela Rosa (widowed by then) sold one-half of the said property to
Juliana Salazar for P95.00. This sale between him and Salazar, though evidenced by a
document, was not registered. Nevertheless, Juliana Salazar constructed a house on the lot she
purchased immediately after the sale. On March 10, 1964, petitioner spouses Diosdado Nuguid
and Marqiueta Venegas (spouses Nuguid) caused the registration of a document entitled
"Kasulatan ng Partihan at Bilihan" (Kasulatan) dated June 6, 1961. In this document, Marciana
dela Rosa, together with the heirs of Victorino and Crisanta dela Rosa, sold to spouses Nuguid
the entire area of the property for the sum of P300.00. Subsequently, OCT No. 3778 was
cancelled by the Register of Deeds of Bataan, and TCT No. T-12782 was issued in the spouses
Nuguid’s names.

Private respondents claimed that the presented by spouses Nuguid was forged. They also
allegedly discovered the forged deed as well as the certificate of title in the name of the
petitioners much later, that is, on February 28, 1978, when respondents Amorita Guevarra and
Teresita Guevarra thought of having the title of their grandmother Juliana Salazar, registered.
On the other hand, spouse Nuguid assert that in the latter part of 1960, Nicolas dela Rosa,
uncle of respondent Marciana dela Rosa and grandfather of the other heirs-signatories, offered
to sell the subject land to them. Apparently, Nicolas dela Rosa claimed that he had already
purchased the shares of the heirs over the subject property as evidenced by a private
document entitled "Kasunduan" (Kasunduan) dated August 31, 1955, and as a matter of fact, he
had in his possession the original certificate of title covering the property in the name of the
deceased Victorino and Crisanta dela Rosa.

The CFI of Bataan dismissed the complaint filed by private respondents, but the Court of
Appeals reversed said decision and ordered the spouses Nuguid to execute a deed of
reconveyance in favor of herein respondents.

ISSUE: Who is the rightful owner of the subject property?

RULING: The Supreme Court reinstated the decision of the CFI of Bataan. The basis for the
Court of Appeals' conclusion that petitioners were buyers in bad faith is ambiguous because
said court relied on the singular circumstance that the petitioners are from Orani, Bataan, and
should have personally known that the private respondents were the persons in actual
possession. However, at the time of the purchase, the spouses Nuguid dealt with Pedro
Guevarra and Pascuala Tolentino, the latter being the actual occupants. The respondents
Guevarras, children of the said Pedro and Pascuala Guevarra, came into the picture only after
their parents died. As for the respondent heirs of Victorino dela Rosa, their being in actual
possession of any portion of the property was, likewise, simply presumed or taken for granted
by the Court of Appeals.

The private respondents cannot also honestly claim that they became aware of the spouses
Nuguid’s title only in 1978, because ever since the latter bought the property in 1961, the
spouse Nuguid have occupied the same openly, publicly, and continuously in the concept of
owners, even building their house thereon. For seventeen years they were in peaceful
possession, with the respondents Guevarras occupying less than one-half of the same
property.
Ballatan vs. Court of Appeals
304 SCRA 37
1999

Facts: Eden Ballatan, together with other petitioners, is living in Lot No. 24. Respondent
Winston Go is living in and registered owners of Lot No. 25 and 26. And Li Ching Yao is living in
and the registered owner of Lot. 27. The Lots are adjacent to each other.

When Ballatan constructed her house in her lot, she noticed that the concrete fence and side
pathway of the adjoining house of respondent Winston Go encroached on the entire length of
the eastern side of her property. She was informed by her contractor of this discrepancy, who
then told respondent Go of the same.

Respondent, however, claims that his house was built within the parameters of his father’s lot;
and that this lot was surveyed by engineer Jose Quedding, the authorized surveyor of Araneta
Institute of Agriculture (AIA). Petitioner called the attention of AIA on the matter and so the
latter authorized another survey of the land by Engineer Quedding. The latter then did the
survey twice which led to the conclusion that Lots Nos 25, 26 and 27 moved westward to the
eastern boundary of Lot 24. It was later on discovered by the courts that Go encroached 42
square meters from the property of Ballatan and Yao encroached 37 square meters on Go’s
property.

Ballatan made written demands to the respondent to dismantle and move their improvements
and since the latter wasn’t answering the petitioner filed accion publiciana in court.

Issue: Whether or not both of the parties acted in good faith.

Ruling: Art 448 is the proper remedy (Lower Courts are wrong in awarding the damages). It was
established in the case that the parties had no knowledge of the encroachment until Ballatan
noticed it there all of them were builders in Good faith. In that scenario they have two options.
1st option is that the land owner will buy the improvements and the 2nd option is to oblige the
builders to buy the land given that the value of the land is not considerably more than the
buildings or tree; otherwise the owner may remove the improvements thereon.

Builder, planter or sower, however, is not obliged to purchase the land if its value is
considerably more than the building, planting or sowing. In such case, the builder, planter or
sower must pay rent to the owner of the land. If the parties cannot come to terms over the
conditions of the lease, the court must fix the terms thereof. The right to choose between
appropriating the improvement or selling the land on which the improvement of the builder,
planter or sower stands, is given to the owner. If the option chooses is to sell the lot, the price
must be fixed at the prevailing market value at the time of payment.
Rules Governing Alluvion

HEIRS OF NAVARRO V. IAC


268 SCRA 74, 85
1997

Facts: on 1946 sinforoso pascual sought to register the land on the northern section of his
existing property. His current registered property is bounded on the east by talisay river, on the
west by bulacan river and on the north by the manila bay. Both rivers flow towards the manila
bay. Because of constantly flowing water, extra land formed in the northern most section of the
property. Pascual claimed that the talisay and bulacan rivers deposited more silt resulting on
accretion. He claimed this land as riprarian owner.

Then a new party, mr emiliano navarro jumped into the fray opposing the same application,
stating the he leased part of the property sought to be registered. He sought to protect his
fishpond that rested on the same property. Sinforoso was not amused and filed ejectment
against mr. Navarro, claiming that navarro used stealth force and strategy to occupy a portion
of his land. Pascual lost the case against navarro so he appealed.

Issue: whether or not the accretion taking place on property adjacent to the sea can be
registered under the torrens system.

Ruling: it cannot be registered. This is land of public domain. Pascual claimed ownership under
article 457 of the civil code saying that the disputed 14-hectare land is an accretion caused by
the joint action of the talisay and bulacan rivers.

Art 457: accretion as a mode of acquiring property and requires the concurrence of the
following requisites: (1) that the accumulation of soil or sediment be gradual and
imperceptible; (2) that it be the result of the action of the waters of the river; and (3) that the
land where the accretion takes place is adjacent to the bank of the river.

Unfortunately, pasucal and heirs claim of ownership based on art 457 is misplaced. If there’s
any land to be claimed, it should be land adjacent to the rivers talisay and bulacan. The law is
clear on this. Accretion of land along the river bank may be registered. This is not the case of
accretion of land on the property adjacent to Manila Bay. Lands added to the shores by
accretions and alluvial deposits caused by the action of the sea, form part of the public domain.
When they are no longer washed by the waters of the sea and are not necessary for purposes
of public utility, or for the establishment of special industries, or for the coast-guard service, the
government shall declare them to be the property of the owners of the estates adjacent
thereto and as increment thereof.
VDA. DE NAZARENO vs. COURT OF APPEALS
G.R. No. 98045
June 26, 1996

Facts: The subject of this controversy is a parcel of land situated in Telegrapo, Puntod, Cagayan
de Oro City. Said land was formed as a result of sawdust dumped into the dried-up Balacanas
Creek and along the banks of the Cagayan river.
Sometime in 1979, private respondents Jose Salasalan and Leo Rabaya leased the subject lots
on which their houses stood from one Antonio Nazareno, petitioners' predecessor-in-interest.

In the latter part of 1982, private respondents allegedly stopped paying rentals. As a result,
Antonio Nazareno and petitioners filed a case for ejectment.

Issue: Whether or not the subject land is public land.

Ruling: The subject land is public land. The court cited the case of: In Republic v. CA, this Court
ruled that the requirement that the deposit should be due to the effect of the current of the
river is indispensable. This excludes from Art. 457 of the Civil Code all deposits caused by
human intervention. Putting it differently, alluvion must be the exclusive work of nature. Thus,
in Tiongco v. Director of Lands, et al., where the land was not formed solely by the natural
effect of the water current of the river bordering said land but is also the consequence of the
direct and deliberate intervention of man, it was deemed a man-made accretion and, as such,
part of the public domain.

In the case at bar, the subject land was the direct result of the dumping of sawdust by the Sun
Valley Lumber Co. consequent to its sawmill operations. Even if this Court were to take into
consideration petitioners' submission that the accretion site was the result of the late Antonio
Nazareno's labor consisting in the dumping of boulders, soil and other filling materials into the
Balacanas Creek and Cagayan River bounding his land, the same would still be part of the public
domain.
ROXAS vs. TUASON
9 Phil. 408

Facts: On February 19, 1906, attorneys Rosado, Sanz & Opisso, on behalf of Pedro P. Roxas,
applied for the registration of the estate owned by the said Roxas, known as the Hacienda de
San Pedro Macati, in accordance with the provisions of the Land Registration Act; said hacienda
was acquired by the petitioner by inheritance under the will of his late father, Jose Bonifacio
Roxas, y Ubaldo.

The property consists of four different parcels of land, irregular shape, designated on the
accompanying plan under the letters "A", "B", "C", and "D", containing a total area of 1,761
hectares 51 ares and 5 centares, equivalent to 17,615,105 square meters, and according to the
last assessment for the purpose of taxation assessed at P415,221.34, of which P59,904
corresponded to the portion of said hacienda included within the limits of the city of Manila
and P256,769 corresponded to that portion situated in the Province of Rozal. The building
constructed of strong materials, called the "Casa-Quinta" or "Casa de Ingenieros," belonging
also to said Roxas, is erected within parcel "C," occupying, together with its appurtenances, an
area of 8,430 square meters, and was assessed at P98,557.34.
It does not appear that said hacienda is mortgaged nor that any person has any right to or any
interest therein; and it is almost wholly occupied at the present time, under lease, by about 429
tenants whose names, residences, and postal addresses, as well as the residence of the owner
of the property and of his attorney in fact, are stated in the application.

Issue: Whether or not the said property is public dominion.

Ruling: Article 366 of the Civil Code states that "The accretions which banks of rivers may
gradually receive from the effects of the currents belong to the owners of the estates bordering
thereon."

The provision in this article is perfectly applicable to the strip of land, which, on account of the
accretion, has come to be undeniable increase in the land of the hacienda inasmuch as it has
increased all along the bank of the creek, the gradual effect of the currents; and even though
the law does not require an express act of possession of the accretion which has enlarged the
estate, it is certain that the owner of the hacienda has possessed it for more than thirty years
through his tenants, who have been cultivating their respective parcels of land together with
the corresponding portion of the said strip down to the bank of said creek.
CUREG vs. INTERMEDIATE APPELLATE COURT
177 SCRA 313
1989

FACTS: In 1982 Apostol, et al filed a complaint for quieting of title and damages with
preliminary injunction against the Carniyans with the RTC of Isabela. Apostol, et al alleged that
they are the legal heirs of Domingo Geraro who has been in open, continuous, exclusive and
notorious possession of a parcel of land referred to as “motherland” since time immemorial.
That said land was declared for taxation purposes in the name of Francisco Gerardo. During the
execution of the extra-judicial partition with voluntary reconveyance, the motherland already
manifested signs of accretion of about 3 hectares on the north caused by the northward
movement of the Cagayan River; that Domingo Apostol declared the motherland and its
accretion for tax purposes under a tax declaration. Apostol et al. were about to cultivate their
“motherland” together with its accretion, they were prevented and threatened by the
Carniyans from continuing to do so.

Carniyans’ alleged that the “motherland” is non-existent. that the "motherland; that Antonio
Carniyan, petitioners' predecessor-in-interest, was the owner of a piece of land bounded on the
north by Cagayan River and not by the land of Francisco Gerardo as claimed by private
respondents; that the "subject land" is an accretion to their registered land and that petitioners
have been in possession and cultivation of the "accretion" for many years now.
In a decision rendered by the trial court held that respondent Domingo Apostol, thru his
predecessors-in-interest had already acquired an imperfect title to the subject land. The
petitioners appealed to the then Intermediate Appellate Court which affirmed the decision of
the trial court.

ISSUE: Whether or not the petitioners can be considered as riparian owners who are entitled to
the subject land which is an accretion to the registered land.

RULING: Yes. Original certificate of title is better than tax declaration. It should be noted that
the herein Apostols’ claim of ownership of their alleged two and a half hectare "motherland" is
anchored mainly on four (4) tax declarations while the Carniyans relied on the indefeasibility
and incontrovertibility of their OCT. The declaration of ownership for purposes of assessment
on the payment of the tax is not sufficient evidence to prove ownership nor proof of the area
covered therein. Since OCT clearly stated that subject land is bounded on the north by the
Cagayan River, Apostol’s claim over their “motherland”, allegedly existing between petitioners’
land and the Cagayan River, is deemed barred and nullified with the issuance of the OCT.

Thus the alleged “motherland” claimed by private respondents is non-existent. The “subject
land” is an alluvial deposit left by the northward movement of the Cagayan River and pursuant
to NCC 457: “to the owners of the land adjoining the banks of river belong the accretion which
they gradually receive from the effects of the current of the waters.”
Change of Course of River

AGNE vs. DIRECTOR OF LANDS


G.R. NO. L-40399
FEBRUARY 6, 1990

FACTS: Petitioners filed a complaint against the respondents Director of Lands and spouses
Agpoon with the former Court of First Instance of Pangasinan for annulment of title,
reconveyance of and/or action to clear title to a parcel of land. Petitioners alleged in their said
complaint that the land in question, which was formerly a portion of the bed of Agno-Chico
river which was abandoned as a result of the big flood in 1920, belongs to them pursuant to the
provision of Article 370 of the old Civil Code; that when respondent spouses filed a complaint
against them, they found out that the said land was granted by the Government to
Herminigildo Agpoon under Free Patent No. 23263, pursuant to which Original Certificate of
Title No. 2370 issued in the latter's name; and that the said patent and subsequent titles issued
pursuant thereto are null and void since the said land, an abandoned river bed, is of private
ownership and, therefore, cannot be the subject of a public land grant.

ISSUE: Whether or not the subsequent derivative certificates of title in question were null and
void ab initio.

RULING: Yes. Private ownership of land is not affected by the issuance of a free patent over the
same land because the Public Land Act applies only to lands of the public domain. Only public
land may be disposed of by the Director of Lands. Since as early as 1920, the land in dispute was
already under the private ownership of herein petitioners and no longer a part of the lands of
the public domain, the same could not have been the subject matter of a free patent. The
patentee and his successors in interest acquired no right or title to the said land. Necessarily,
Free Patent No. 23263 issued to Herminigildo Agpoon is null and void and the subsequent titles
issued pursuant thereto cannot become final and indefeasible. A free patent which purports to
convey land to which the Government did not have any title at the time of its issuance does not
vest any title in the patentee as against the true owner. The Court has previously held that the
Land Registration Act and the Cadastral Act do not give anybody who resorts to the provisions
thereof a better title than what he really and lawfully has.
Quieting of Title

Requisites

CHUNG, ET AL. VS. MONDRAGON


G.R. NO. 179754
NOVEMBER 21, 2012

FACTS: Petitioners Joaquin G. Chung, Jr., Paz Royeras-Soler, and Mansueto Maceda are
descendants of Rafael Mondragon (Rafael) by his first wife, Eleuteria Calunia (Eleuteria), while
respondent Jack Daniel Mondragon (Jack Daniel) is Rafaels descendant by his second wife,
Andrea Baldos (Andrea).

OCT No. 22447 is registered in the name of "Heirs of Andrea Baldos represented by Teofila G.
Maceda" and covers 16,177 square meters of land in Macrohon, Southern Leyte (the land).
Chung, et al. claim that from 1921 up to 2000, Rafael appeared as owner of the land in its tax
declaration, and that a free patent was issued in 1987 in the name of Andreas heirs upon
application of Teofila G.Maceda (Teofila), who is petitioners sister.

On the other hand, respondents Bourbon, et al. claim that Andrea is the exclusive owner of the
land, having inherited the same from her father and that after Andrea died, his son Fortunato
Mondragon inherited the land; and when the latter died, his son Jack Daniel (herein
respondent) came into possession and enjoyment thereof. Sometime in the year 2000, Jack
Daniel sold a 1,500-square meter portion of the land to his co-respondent Clarinda Regis-
Schmitz .

On the claim that Jack Daniel had no right to sell a portion of the land and that the sale to Regis-
Schmitz created a cloud upon their title, Chung, Jr., et al. filed an action to quiet title. The RTC
dismissed the complaint of Chung, Jr., et al. The CA sustained the trial court.

ISSUE: Whether or not the action to quiet title should prosper?

RULING: The petition lacks merit. The issues in a case for quieting of title are fairly simple; the
plaintiff need to prove only two things, namely: "(1) the plaintiff or complainant has a legal or
an equitable title to or interest in the real property subject of the action; and (2) that the deed,
claim, encumbrance or proceeding claimed to be casting a cloud on his title must be shown to
be in fact invalid or inoperative despite its prima facie appearance of validity or legal efficacy.

It is evident from the title that the land belongs to no other than the heirs of Andrea Baldos,
Rafaels second wife. The land could not have belonged to Rafael, because he is not even named
in OCT No. 22447.With greater reason may it be said that the land could not belong to
petitioners, who are Rafaels children by his first wife Eleuteria. Unless Eleuteria and Andrea
were related by blood such fact is not borne out by the record they could not be heirs to each
other. Add to this is the fact that petitioners are not in possession of the land. Petitioners do
not possess legal or equitable title to the land.
BAHA'IS VS. PASCUAL
G.R. NO. 169272
JULY 11, 2012

FACTS: December 11, 2000, Bahais filed a complaint with RTC for quieting of title, injunction
and other claims against Silverio Songcuan and/or his heirs, the secretary of DENR and the
regional executive director of DENR of Tuguegarao, Cagayan.

Bahais alleged that it is the lawful and absolute owner of two parcels of land, who acquired
ownership from Marcelina Ordono. The Bahais had been in possession of the land for 30 years,
until the Bureau of Lands rejected the sales applications of the Bahais predecessors-in-interest
for the lots and ordered all those in privity to vacate the lots and to remove their
improvements. DENR secretary affirmed with this decision. Recourse to the office of the
President had been unavailing, so DENR issued writs of execution pursuant to the President's
decision.

Pascual moved to dismiss the complaint for failure to state cause of action. Saying that the
petitioner had no legal right to file the complaint since the final and executory Bureau of Lands’
decision ruled that the petitioner was not entitled to possess the lots.

RTC's Ruling: denied the motion to dismiss, Bureau of Lands was not yet final since President's
ruling on the appeal was unavailable. Respondent elevated this case to the CA, questioning the
propriety of the RTC's denial of the motion to dismiss.

ISSUE: Whether CA committed a reversible error in finding that RTC committed a grave abuse
of discretion in not dismissing the complaint.

RULING: Petition lacks merit, CA committed no reversible error. A cause of action is the act or
omission by which a party violates a right of another. A complaint states a cause of action when
it contains three essential elements: (1) a right in favor of the plaintiff by whatever means and
whatever law it arises; (2) the correlative obligation of the defendant to respect such right; and
(3) the act or omission of the defendant violates the right of the plaintiff. If any of these
elements is absent, the complaint becomes vulnerable to a motion to dismiss on the ground of
failure to state a cause of action.

"Failure to state a cause of action refers to the insufficiency of allegation in the pleading. In
resolving a motion to dismiss based on the failure to state a cause of action only the facts
alleged in the complaint must be considered. The test is whether the court can render a valid
judgment on the complaint based on the facts alleged and the prayer asked for."

From these allegations, we find it clear that the petitioner no longer had any legal or equitable
title to or interest in the lots. The petitioner’s status as possessor and owner of the lots had
been settled in the final and executory December 4, 1985 decision of the Bureau of Lands that
the DENR Secretary and the OP affirmed on appeal. Thus, the petitioner is not entitled to the
possession and ownership of the lots.
VDA DE AVILES V. COURT OF APPEALS
264 SCRA 473

FACTS: Petitioners aver that they are the actual possessors of a parcel of land, bounded on the
N by Camilo Aviles; on the E by Malawa River, on the S by Anastacio Aviles and on the W by
Juana and Apolonio Joaquin. This property is the share of their father, Eduardo Aviles and
brother of the defendant, in the estate of their deceased parents.

Eduardo Aviles was in actual possession property since 1957. In fact, the latter mortgaged the
same with the bank. When the property was inspected by a bank representative, Eduardo
Aviles, in the presence of the boundary owners, namely, defendant Camilo Aviles, Anastacio
Aviles and Juana and Apolonio Joaquin pointed to the inspector the existing earthen dikes as
the boundary limits of the property and nobody objected. When the real estate mortgage was
foreclosed, the property was sold at public auction but this was redeemed by plaintiffs’ mother
and the land was subsequently transferred and declared in her name.

Defendant Camilo Aviles asserted a color of title over the northern portion of the property by
constructing a bamboo fence (thereon) and moving the earthen dikes.

Defendant Camilo Aviles admitted the agreement of partition executed by him and his
brothers, Anastacio and Eduardo. The respective area(s) allotted to them was agreed and
measured before the execution of the agreement. At present, he is only occupying a smaller
than his actual share.

The trial court disposed of the case thus ordering the parties to employ the services of a Land
Surveyor of the Bureau of Lands to relocate and determine the extent and the boundary limit of
the land of the defendant on its southern side in order that the actual area given to the
defendant be determined.

Dissatisfied with the trial court’s decision, petitioners appealed to the respondent appellate
Court. The Court of Appeals affirmed in part the decision of the trial court, reasoning that a
special civil action for quieting of title is not the proper remedy for settling a boundary dispute,
and that petitioners should have instituted an ejectment suit instead.

ISSUE: Whether or not the complaint for quieting of title is the proper remedy for settling
boundary dispute.

RULING: The Supreme Court ruled that Quieting of Title Not Proper Remedy for Settling
Boundary Dispute. Quieting of title is a common law remedy for the removal of any cloud upon
or doubt or uncertainty with respect to title to real property.

The Civil Code authorizes the said remedy in the following language:

“Art. 476. Whenever there is a cloud on title to real property or any interest therein, by reason
of any instrument, record, claim, encumbrance or proceeding which is apparently valid or
effective but is, in truth and in fact, invalid, ineffective, voidable, or unenforceable, and may be
prejudicial to said title, an action may be brought to remove such cloud or to quiet the title.

An action may also be brought to prevent a cloud from being cast upon a title to real property
of any interest therein."

In fine, to avail the remedy of quieting of title, a plaintiff must show that there is an instrument,
record, claim, encumbrance or proceeding which constitutes or casts a cloud, doubt, question
or shadow upon the owner’s title to or interest in real property.

As correctly held by the respondent Court, both plaintiffs and defendant admitted the existence
of the agreement of partition and in accordance therewith, a fixed area was allotted to them
and that the only controversy is whether these lands were properly measured. There is no
adverse claim by the defendant “which is apparently valid, but is, in truth and in fact, invalid,
ineffective, voidable, or unenforceable” and which constitutes a cloud thereon.
Corollarily, and equally as clear, the construction of the bamboo fence enclosing the disputed
property and the moving of earthen dikes are not the “clouds” or “doubts” which can be
removed in an action for quieting of title.
CO-OWNERSHIP

Nature of Co-ownership

ALEJANDRINO VS. COURT OF APPEALS


G.R. NO. 114151
SEPTEMBER 17, 1998

FACTS: The late spouses Alejandrino left their six children a lot in Cebu City. Upon the death of
the spouses, the property should have been divided among their children, however, the
estate of the Alejandrino spouses was not settled in accordance with the procedures. Petitioner
Mauricia (one of the children) allegedly purchased portion of the lots from her brothers,
Gregorio’s, Ciriaco’s and Abundio’s share. It turned out, however, that a third party named
Nique, the private respondent in this case, also purchased portions of the property from
Laurencia, Abundio and Marcelino. However, Laurencia (the alleged seller to Nique) later
questioned the sale in an action for quieting of title and damages. The trial court (Quieting of
title case) ruled in favor of Nique and declared him the owner of the lots. Laurencia appealed
the decision to the Court of Appeals but later withdrew the same.
Nique filed a motion for the segregation of the portion of the property that had been declared
by the trial court (Quieting of title case) as his own by virtue of purchase. The trial court
segregated the property on the basis of the Extra-Judicial Settlement between Mauricia and
Laurencia.

ISSUE: Whether or not a co-owner may validly sell specific portions of unpartitioned property to
a third party.

RULING: Yes. A co-owner is entitled to sell his undivided share. A sale of the entire property by
one co-owner without the consent of the other co-owners is not null and void. However, only
the rights of the co-owner-seller are transferred, thereby making the buyer a co-owner of the
property.( Bailon-Casilao vs. Court of Appeals)

Article 1078 of the Civil Code provides that where there are two or more heirs, the whole estate
of the decedent is, before partition, owned in common by such heirs, subject to the payment of
the debts of the deceased. Under a co-ownership, the ownership of an undivided thing or right
belongs to different persons. Each co-owner of property which is held pro indiviso exercises his
rights over the whole property and may use and enjoy the same with no other limitation than
that he shall not injure the interests of his co-owners. The underlying rationale is that until a
division is made, the respective share of each cannot be determined and every co-owner
exercises, together with his co-participants, joint ownership over the pro indiviso property, in
addition to his use and enjoyment of the same.

Although the right of an heir over the property of the decedent is inchoate as long as the estate
has not been fully settled and partitioned,the law allows a co-owner to exercise rights of
ownership over such inchoate right. Thus, the Civil Code provides:

ART. 493. Each co-owner shall have the full ownership of his part and of the fruits and benefits
pertaining thereto, and he may therefore alienate, assign or mortgage it, and even substitute
another person in its enjoyment, except when personal rights are involved. But the effect of the
alienation or the mortgage, with respect to the co-owners, shall be limited to the portion which
may be allotted to him in the division upon the termination of the co-ownership.
With respect to properties shared in common by virtue of inheritance, alienation of a pro
indiviso portion thereof is specifically governed by Article 1088 that provides:

ART. 1088. Should any of the heirs sell his hereditary rights to a stranger before the partition,
any or all of the co-heirs may be subrogated to the rights of the purchaser by reimbursing him
for the price of the sale, provided they do so within the period of one month from the time they
were notified in writing of the sale by the vendor.

In the instant case, Laurencia was within her hereditary rights in selling her pro indiviso share in
Lot No. 2798. However, because the property had not yet been partitioned in accordance with
the Rules of Court, no particular portion of the property could be identified as yet and
delineated as the object of the sale. Thus, interpreting Article 493 of the Civil Code providing
that an alienation of a co-owned property shall be limited to the portion which may be allotted
to (the seller) in the division upon the termination of the co-ownership, the Court said: x x x
(p)ursuant to this law, a co-owner has the right to alienate his pro-indiviso share in the co-
owned property even without the consent of the other co-owners. Nevertheless, as a mere part
owner, he cannot alienate the shares of the other co-owners.
RULES GOVERNING CO-OWNERSHIP

DE GUIA vs. COURT OF APPEALS


413 SCRA 114, 124
2003

FACTS: On 12 May 1986, ABEJO instituted an action for recovery of possession with damages
against DE GUIA. In his complaint, ABEJO alleged that he is the owner of the undivided portion
of a property used as a fishpond. He alleged ownership over approximately 39,611 square
meters out of the FISHPONDs total area of 79,220 square meters. ABEJO further averred that
DE GUIA continues to possess and use the FISHPOND without any contract and without paying
rent to ABEJOs damage and prejudice. ABEJO also complained that DE GUIA refuses to
surrender ownership and possession of the FISHPOND despite repeated demands to do so after
DE GUIAs sublease contract over the FISHPOND had expired. ABEJO asked the trial court to
order DE GUIA to vacate an approximate area of 39,611 square meters as well as pay damages.
The trial court rendered its decision in favor of plaintiff ABEJO and against the defendant
DEGUIA. Aggrieved, DE GUIA went to the Court of Appeals insisting the trial court erred in
ordering him to vacate and surrender possession of the undivided portion of the FISHPOND.
The Court of Appeals found DE GUIA’s appeal without merit and affirmed the trial court’s
decision. Hence, this petition.

ISSUE: Whether or not a co-owner can be excluded from a specific portion of an undivided
property?

RULING: No. Under Article 484 of the Civil Code, there is co-ownership whenever the
ownership of an undivided thing or right belongs to different persons. A co-owner of an
undivided parcel of land is an owner of the whole, and over the whole he exercises the right of
dominion, but he is at the same time the owner of a portion which is truly abstract. On the
other hand, there is no co-ownership when the different portions owned by different people
are already concretely determined and separately identifiable, even if not yet technically
described.

Article 487 of the Civil Code provides, any one of the co-owners may bring an action in
ejectment. This article covers all kinds of actions for the recovery of possession. Article 487
includes forcible entry and unlawful detainer (accion interdictal), recovery of possession (accion
publiciana), and recovery of ownership (accion de reivindicacion). The summary actions of
forcible entry and unlawful detainer seek the recovery of physical possession only. These
actions are brought before municipal trial courts within one year from dispossession. However,
accion publiciana, which is a plenary action for recovery of the right to possess, falls under the
jurisdiction of the proper regional trial court when the dispossession has lasted for more than
one year. Accion de reivindicacion, which seeks the recovery of ownership, also falls under the
jurisdiction of the proper regional trial court.
Any co-owner may file an action under Article 487 not only against a third person, but also
against another co-owner who takes exclusive possession and asserts exclusive ownership of
the property. In the latter case, however, the only purpose of the action is to obtain recognition
of the co-ownership. The plaintiff cannot seek exclusion of the defendant from the property
because as co-owner he has a right of possession. The plaintiff cannot recover any material or
determinate part of the property.

Following the inherent and peculiar features of co-ownership, while ABEJO and DE GUIA have
equal shares in the FISHPOND quantitatively speaking, they have the same right in a qualitative
sense as co-owners. Simply stated, ABEJO and DE GUIA are owners of the whole and over the
whole, they exercise the right of dominion. However, they are at the same time individual
owners of a portion, which is truly abstract because until there is partition, such portion
remains indeterminate or unidentified. As co-owners, ABEJO and DE GUIA may jointly exercise
the right of dominion over the entire FISHPOND until they partition the FISHPOND by
identifying or segregating their respective portions.

We rule that a co-owner may file an action for recovery of possession against a co-owner who
takes exclusive possession of the entire co-owned property. However, the only effect of such
action is a recognition of the co-ownership. The courts cannot proceed with the actual
partitioning of the co-owned property. Thus, judicial or extra-judicial partition is necessary to
effect physical division of the FISHPOND between ABEJO and DE GUIA. An action for partition is
also the proper forum for accounting the profits received by DE GUIA from the FISHPOND.
However, as a necessary consequence of such recognition, ABEJO shall exercise an equal right
to possess, use and enjoy the entire FISHPOND.
BAILON-CASILAO VS. CPURT OF APPEALS
160 SCRA 738, 745
APRIL 15, 1988

FACTS: Rosalia, Gaudencio, Sabina, Bernabe, Nenita and Delia Bailon are co-owners of a parcel
of land with an area of 48,849 square meters, with a 1/6 share each. On August 23, 1948,
Rosalia Bailon and Gaudencio Bailon sold a portion of the said land consisting of 16,283 square
meters to Donato Delgado. On May 13, 1949, Rosalia Bailon alone sold the remainder of the
land consisting of 32,566 square meters to Ponciana V. Aresgado de Lanuza. On the same date,
Lanuza acquired from Delgado the 16,283 square meters of land which the latter had earlier
acquired from Rosalia and Gaudencio. On December 3, 1975, John Lanuza, acting under a
special power of attorney given by his wife, Ponciana V. Aresgado de Lanuza, sold the two
parcels of land to Celestino Afable, Sr.
The lower court rendered a decision declaring Celestino Afable, a co-owner of the land
described in paragraph III of the complaint having validly bought the two-sixth (2/6) respective
undivided shares of Rosalia Bailon and Gaudencio Bailon. The court also declaring Sabina,
Bernabe, Delia and the heirs of Nebita as pro-indiviso co-owners, having 1/6 share each, of the
property described in paragraph III of the complaint.

ISSUE: Whether or not a sale by one or more co-owners of the entire property held in common
without the consent of all the co-owners is valid.

RULING: Yes. The rights of a co-owner of a certain property are clearly specified in Article 493
of the Civil Code which states that each co-owner shall have the full ownership of his part and
of the acts and benefits pertaining thereto, and he may therefore alienate assign or mortgage it
and even substitute another person in its enjoyment, except when personal rights are involved.
But the effect of the alienation or mortgage, with respect to the co-owners, shall be limited to
the portion which may be allotted to him in the division upon the termination of the co-
ownership.

As early as 1923, this Court has ruled that even if a co-owner sells the whole property as his, the
sale will affect only his own share but not those of the other co-owners who did not consent to
the sale. This is because under the aforementioned codal provision, the sale or other
disposition affects only his undivided share and the transferee gets only what would
correspond to his grantor in the partition of the thing owned in common. Consequently, by
virtue of the sales made by Rosalia and Gaudencio Bailon which are valid with respect to their
proportionate shares, and the subsequent transfers which culminated in the sale to private
respondent Celestino Afable, the said Afable thereby became a co-owner of the disputed parcel
of land.

A co-owner is entitled to sell his undivided share, a sale of the entire property by one co-owner
without the consent of the other co-owners is not null and void.
GAPACAN vs. OMIPET
387 SCRA 383

FACTS: Paicat Gapacan is the primitive possessor of an unregistered land in Mt. Province,
divided into 3 parcels of riceland and another planted to camote and declared by him for
taxation purposes. He had two children Maria and Antonio. Antonio left for a long while to try
his luck in the mines Benguet. Maria remained, took care of their father and eventually took
over the cultivation of the land.

Antonio Gapacan returned to and executed an Affidavit of Transfer of Real Property showing
that the property had been transferred to him by his sister Maria Gapacan-Omipet (Omipet)
making him in effect the legal owner of the property in question. Since then, Antonio Gapacan’s
family (Gapacans) had been occupying and cultivating the property.
Sometime in 1992, Omipet hired laborers to clear and cultivate portions of the disputed
property. Gapacans prohibited them Gapacans and ordered the defendants to vacate the land
and restore possession to plaintiffs.

Omipet then filed an action to quiet title in RTC and that she be declared the lawful owner. RTC
adjudged that Gapacans have right of possession over the land. On appeal CA, declared that the
land is common property of both Omipet and Gapacans and ordered its partition.

ISSUE: Whether or not the petitioner and defendant are co-owners of the disputed property.

RULING: Yes. Although the evidence preponderates in favor of Antonio Gapacan and
subsequently his heirs upon his death and it has been clearly established that Antonio and his
family had been in possession of the subject realty since 1971, Antonio could not honestly claim
the rights of a possessor in good faith since his tax declarations, and more so, his Affidavit of
Transfer of Real Property, were either spurious or founded on false and unlawful claims. The
parcels of land in question, as part of the hereditaments of Paicat, a common ancestor of Maria
and Antonio, were given to neither of them in particular. It is difficult to believe that Maria and
Antonio were blissfully ignorant of their respective legal rights over the disputed realty. As the
two (2) surviving heirs of the Paicat Gapacan, neither Maria nor Antonio can claim absolute
ownership over the entire property to the prejudice of the other, for each, in legal
contemplation, is entitled to only one-half (1/2) pro-indiviso share of his or her father's estate.
Prior to partition, Maria and Antonio, and upon the latter's death, the petitioners, hold the
disputed property in their capacity as co-owners.

The juridical concept of co-ownership is unity of the object or property and plurality of subjects.
Each co-owner, jointly with the other co-owners, is the owner of the whole property, but at the
same time of the undivided aliquot part. Each co-owner has the right to sell, assign or dispose
of his share, unless personal rights are involved. He may also lose such rights to others, as by
prescription thereof by a co-owner.
EXTINGUISHMENT of CO-OWNERSHIP

ADILLE vs. COURT OF APPEALS,


157 SCRA 455
JANUARY 29, 1988

FACTS: Felisa Alzul owned a lot with an area of some 11,325 sq. m. She married twice in her
lifetime; the first, with one Bernabe Adille, with whom she had as an only child, herein
defendant Rustico Adille; in her second marriage with one Procopio Asejo, her children were
herein plaintiffs, — now, sometime in 1939, said Felisa sold the property in pacto de retro to
certain 3rd persons, period of repurchase being 3 years, but she died in 1942 without being
able to redeem and after her death, but during the period of redemption, herein defendant
repurchased, by himself alone, and after that, he executed a deed of extra-judicial partition
representing himself to be the only heir and child of his mother Felisa with the consequence
that he was able to secure title in his name alone also, so that OCT. No. 21137 in the name of
his mother was transferred to his name, that was in 1955.

ISSUE: Whether or not a co-owner may acquire exclusive ownership over the property held in
common.

RULING: No. While the records show that the petitioner redeemed the property in its entirety,
shouldering the expenses therefore, that did not make him the owner of all of it. In other
words, it did not put to end the existing state of co-ownership. The property remains to be in a
condition of co-ownership. While a vendee a retro, under Article 1613 of the Code, "may not be
compelled to consent to a partial redemption," the redemption by one co-heir or co-owner of
the property in its totality does not vest in him ownership over it. Failure on the part of all the
co-owners to redeem it entitles the vendee a retro to retain the property and consolidate title
thereto in his name. But the provision does not give to the redeeming co-owner the right to the
entire property. It does not provide for a mode of terminating a co-ownership.

Neither does the fact that the petitioner had succeeded in securing title over the parcel in his
name terminate the existing co-ownership. While his half-brothers and sisters are liable to him
for reimbursement as and for their shares in redemption expenses, he cannot claim exclusive
right to the property owned in common. Registration of property is not a means of acquiring
ownership. It operates as a mere notice of existing title, that is, if there is one.
SANCHEZ V. COURT OF APPEALS,
404 SCRA 541, 548
JUNE 20, 2003

FACTS: Lilia Sanchez, petitioner, constructed a house on a lot owned by her parents-in-law. The
lot was registered under TCT together with her co-owners. The lot was registered under TCT in
the name of private respondent Virginia Teria by virtue of a Deed of Absolute Sale by all six (6)
co-owners in her favor. Petitioner claimed that she did not affix her signature on the document
and subsequently refused to vacate the lot, thus prompting private respondent to file an action
for recovery of possession of the aforesaid lot with the Metropolitan Trial Court (MeTC) of
Caloocan City. The MeTC of Caloocan City ruled in favor of private respondent declaring that
the sale was valid only to the extent of 5/6 of the lot and the other 1/6 remaining as the
property of petitioner, on account of her signature in the Deed of Absolute Sale having been
established as a forgery.

Petitioner then elevated her appeal to the Regional Trial Court of Caloocan City, which ordered
the parties to file their respective memoranda of appeal. Counsel for petitioner did not comply
with this order, nor even inform her of the developments in her case. Petitioner not having filed
any pleading with the RTC of Caloocan City, the trial court affirmed decision of the MeTC.

The MeTC issued an order for the issuance of a writ of execution in favor of private
respondent. On November 4, 1999, a Notice to Vacate was served by the sheriff upon
petitioner who however refused to heed the Notice. Private respondent started demolishing
petitioner’s house without any special permit of demolition from the court. Due to the
demolition of her house, petitioner filed her Petition for Relief from Judgment with the RTC on
the ground that she was not bound by the inaction of her counsel who failed to submit
petitioners appeal memorandum. However the RTC denied the Petition and the subsequent
Motion for Reconsideration.
Petitioner filed her petition for Certiorari with the Court of Appeals alleging grave abuse of
discretion on the part of the court a quo. The appellate court dismissed the petition for lack of
merit. Petitioner then filed a Motion for Reconsideration but the Court of Appeals denied the
motion.

ISSUE: Whether or not the Court of Appeals committed grave abuse of discretion in dismissing
the challenged case before it.

RULING: As a matter of policy, the original jurisdiction of the Supreme Court to issue the so-
called extraordinary writs should generally be exercised relative to actions or proceedings
before the Court of Appeals or before constitutional or other tribunals or agencies the acts of
which for some reason or other are not controllable by the Court of Appeals. Where the
issuance of the extraordinary writ is also within the competence of the Court of Appeals or the
Regional Trial Court, it is either of these courts that the specific action for the procurement of
the writ must be presented.

The Rules of Court should be liberally construed in order to promote their object of securing a
just, speedy and inexpensive disposition of every action or proceeding. The rules of procedure
should be viewed as mere tools designed to aid the courts in the speedy, just and inexpensive
determination of the cases before them. Liberal construction of the rules and the pleadings is
the controlling principle to effect substantial justice.
Verily, the negligence of petitioners counsel cannot be deemed as negligence of petitioner
herself. A notice to a lawyer who appears to have been unconscionably irresponsible cannot be
considered as notice to his client. Under the peculiar circumstances of this case, it appears from
the records that counsel was negligent in not adequately protecting his client’s interest, which
necessarily calls for a liberal construction of the Rules.
This case overlooks a basic yet significant principle of civil law: co-ownership. Throughout the
proceedings from the MeTC to the Court of Appeals, the notion of co-ownership was not
sufficiently dealt with. Certiorari should therefore be granted to cure this grave abuse of
discretion. In co-ownership, the relationship of such co-owner to the other co-owners is
fiduciary in character and attribute. Whether established by law or by agreement of the co-
owners, the property or thing held pro-indiviso is impressed with a fiducial nature so that each
co-owner becomes a trustee for the benefit of his co-owners and he may not do any act
prejudicial to the interest of his co-owners.
Before the partition of a land or thing held in common, no individual or co-owner can claim title
to any definite portion thereof. All that the co-owner has is an ideal or abstract quota or
proportionate share in the entire land or thing. Although assigned an aliquot but abstract part
of the property, the metes and bounds of petitioner’s lot has not been designated. As she was
not a party to the Deed of Absolute Sale voluntarily entered into by the other co-owners, her
right to 1/6 of the property must be respected. Partition needs to be effected to protect her
right to her definite share and determine the boundaries of her property. Such partition must
be done without prejudice to the rights of private respondent as buyer of the 5/6 portion of the
lot under dispute.
PANGAN VS. COURT OF APPEALS,
166 SCRA 375, 382
OCT.17, 1988

FACTS: The petitioners filed an application for the registration of the land situated in San
Pascual, Obando, Bulacan in their names by virtue of their continuous and exclusive possession
thereof since 1895, by themselves and their father and grandfather before them. The trial court
issued an order of general default, there being no opposition to the application, and proceeded
to hear the evidence of the applicants ex-parte. The application was then approved. The herein
private respondent filed a petition to set aside the said decision, which the trial Court granted,
admitting at the same time her opposition to the application and setting the case for reception
of her evidence. This evidence sought to show that the land was inherited by Leon Hilario's
three children, but the son, waived his right thereto and thereby made his two sisters, Silvestra
and Catalina, its exclusive co-owners. As Catalina's daughter, she was entitled to one-half of the
property, the other half going to Silvestra's heirs, the petitioners herein and the latter's
grandchildren.

The trial court judge issued an order dismissing the opposition and reinstating his original order
stating that whatever rights the private respondent Teodora might have had over the property
had been forfeited by extinctive prescription because she had left the land in 1942 and had not
since then asserted any claim thereto until 1966. On appeal to the court of appeals, the
decision of the lower court was reversed on the ground that the appellees had not clearly
proved that they had acquired the property by prescription. Hence, the appellant was entitled
to one-half of the property as heir.

ISSUES: Whether or not Teodora Garcia, by her failure to assert her right, allowed the statutory
period to lapse, thus enabling the petitioners to perfect their claim of ownership by acquisitive
prescription and so exclude her from her share in the subject property.

RULING:

It is a settled rule that possession by one co-owner will not be regarded as adverse to the other
co-owners but in fact as beneficial to all of them. Hence, as long as his co-ownership is
recognized, an action to compel partition will not prescribe and may be filed at any time against
the actual possessor by any of the other co-owners. However, if the co-owner actually holding
the property asserts exclusive dominion over it against the other co-owners, the corollary of the
rule is that he can acquire sole title to it after the lapse of the prescribed prescriptive period.

The established evidence clearly shows that the subject land was inherited by the petitioners
and the private respondent as co-heirs, whose possession they continued to acquire
prescriptive title over the property. That possession was originally in the name of all the heirs,
including Teodora Garcia. The petitioners have not proved that their possession excluded their
co-owner and aunt. The petitioners appear to have arrogated the entire property to themselves
upon their father's death when they sought to register the land in their names to the exclusion
of Teodora Garcia. Manifestly, the petitioners have acted in bad faith in denying their aunt and
co-heir her legal share to the property they had all inherited. The decision of the Court of
Appeals has been.

HEIRS OF FLORES RESTAR vs. HEIRS OF DOLORES R. CICHON


475 SCRA 73
2005
FACTS: Emilio Restar died intestate, leaving eight children-compulsory heirs. Restar’s eldest
child, Flores, on the basis of a Joint Affidavit he executed with Helen Restar, caused
the cancellation of Tax Declaration in Emilio Restar’s name. The same covers a 5,918 square
meter parcel of land in Aklan which was among the properties left by Restar. Flores thereafter
sought the issuance of another Tax Declaration in his name. Flores later on died.
Ten years later, the heirs of Flores’ sisters, Dolores R. Cichon, et. al. (Heirs of Cichon) filed a
Complaint against Flores’ heirs for “partition of the lot, declaration of nullity of documents,
ownership with damages and preliminary injunction” before the Regional Trial Court (RTC) of
Aklan alleging that the widow Esmenia appealed to them to allow her to hold on to the lot to
finance the education of her children, to which they agreed on the condition that after the
children had finished their education, it would be divided into eight equal parts; and upon their
demand for partition of the lot, the defendants Flores‘ heirs refused, they claiming that they
were the lawful owners thereof as they had inherited it from Flores. Flores‘ heirs claimed that
they had been in possession of the lot in the concept of owner for more than thirty (30) years
and have been paying realty taxes since time immemorial. And they denied having shared with
the plaintiffs the produce of the lot or that upon Flores’ death in 1989, Esmenia requested the
plaintiffs to allow her to hold on to it to finance her children’s education, they contending that
by 1977, the children had already finished their respective courses.

The RTC of Kalibo, Aklan held that Flores and his heirs had performed acts sufficient to
constitute repudiation of the co-ownership, concluded that they had acquired the lot by
prescription. The Court of Appeals reversed the decision finding that there was no adequate
notice by Flores to his co-heirs of the repudiation of the co-ownership and neither was there a
categorical assertion by the defendants of their exclusive right to the entire lot that barred the
plaintiffs’ claim of ownership.

ISSUE: Whether or not Heirs of Flores acquired ownership over the lot by extraordinary
prescription

RULING: Yes. Acquisitive prescription of dominion and other real rights may be ordinary or
extraordinary. Ordinary acquisitive prescription requires possession of things in good faith and
with just title for a period of ten years. Without good faith and just title, acquisitive prescription
can only be extraordinary in character which requires uninterrupted adverse possession for
thirty years.

When Restar died in 1935, his eight children became pro indiviso co-owners of the lot by
intestate succession. Heirs of Chichon never possessed the lot, however, much less asserted
their claim thereto until January 21, 1999 when they filed the complaint for partition subject of
the present petition. In contrast, Flores took possession of the lot after Restar’s death and
exercised acts of dominion thereon — tilling and cultivating the land, introducing
improvements, and enjoying the produce thereof. Flores’ possession thus ripened into
ownership through acquisitive prescription after the lapse of thirty years in accordance with the
earlier quoted Article 1137 of the New Civil Code.

Heirs of Cichon did not deny that aside from the verbal partition of one parcel of land in
Carugdog, Lezo, Aklan way back in 1945, they also had an amicable partition of the lands of
Emilio Restar in Cerrudo and Palale, Banga Aklan on September 28, 1973. Indeed, the following
acts of Flores show possession adverse to his co-heirs: (1) the cancellation of the tax declaration
certificate in the name of Restar and securing another in his name; (2) the execution of a
Joint Affidavit stating that he is the owner and possessor thereof to the exclusion of
respondents; (3) payment of real estate tax and irrigation fees without respondents having ever
contributed any share therein; and (4) continued enjoyment of the property and its produce to
the exclusion of respondents. And Flores’ adverse possession was continued by his heirs.
The trial court’s finding and conclusion that Flores and his heirs had for more than 38 years
possessed the land in open, adverse and continuous possession in the concept of owner —
which length of possession had never been questioned, rebutted or disputed by any of the heirs
of Cichon, being thus duly supported by substantial evidence, he and his heirs have become
owner of the lot by extraordinary prescription. It is unfortunate that respondents slept on their
rights.
DELIMA vs. COURT OF APPEALS
G.R. NO. L-46296
SEPTEMBER 24, 1991

FACTS: Lino Delima acquired Lot. No. 7758 of the Talisay-Minglanilla Friar Lands Estate in Cebu
by sale on installments from the government. After his demise in 1921 he had his three
brothers and a sister listed as his heirs. The heirs were Eulalio Delima, Juanita Delima, Galileo
Delima, and Vicente Delima. Anew Transfer Certificate of Title was issued in the name of the
Legal Heirs of Lino Delima represented by Galileo Delima. On September 22, 1953, Galileo
executed an affidavit of Extra-judicial Declaration of Heirs adjudicating to himself the subject
property excluding the other heirs. He declared the lot to be of his own and paid for its taxes.
On February 29, 1968, the surviving heirs of Eulalio and Juanita Delima,filed with the Court of
First Instance of Cebu an action for conveyance and partition of property and for the annulment
of the certificate of title issued plus damages against their Uncle Galileo. Vicente Delima was
also later included as party defendant for his refusal to help in the action.

The trial court decided in favor of the petitioners rendering the TCT No. 3009 null and void and
declaring Vicente, the Heirs of Juanita, the Heirs of Eulalio and the Heirs of Galileo to be owners
of the property, each sharing a pro-indiviso share of one-fourth of the whole. The respondents,
Heirs of Galileo Delima, appealed to the Court of Appeals which reversed the decision in their
favor. It upheld the claim of Galileo that the other brothers and sisters have already waived
their rights to the property being that it was Galileo alone that paid for the balance of
the purchase price and the realty taxes for the property.

ISSUE: Whether or not petitioners’ action for partition is already barred by the statutory period
provided by law which shall enable Galileo Delima to perfect his claim of ownership by
acquisitive prescription to the exclusion of petitioners from their shared in the disputed
property?

RULING: Yes, Article 494 (5) of the Civil Code provides that: “No prescription shall run in favor
of a co-owner or co-heir against his co-owners or co-heirs so long as he expressly or impliedly
recognized the co-ownership”.

By this it is therefore understood that possession by a co-owner will not be presumed to be


adverse to the others, but will be held to benefit all. Being that Galileo was holding the property
in representation of the co-owners; he was therefore acting as an administrator who took care
of the property yet still having the ultimate obligation to deliver the property to his co-owners.

However this rule shall no longer apply when one of the co-owners begin to claim the absolute
and exclusive ownership and denies the others any share therein. The imprescriptability of the
action for partition shall no longer apply since Galileo is adversely claiming lone ownership over
the property. In order that a possession be considered adverse amounting to a repudiation of
the co-ownership, the following elements must concur: (1) that the trustee has performed the
unequivocal acts amounting to a repudiation of ownership ,the following must concur: 1) that
the trustee has performed the unequivocal acts amounting to ouster of cestui cui trust;2) that
such positive acts of repudiation has made known to the cestui cui trust and 3) that the
evidence thereon is should be clear and conclusive.

Since Galileo, having executed a deed of partition and obtained subsequent to that the
cancellation of the old title and the creation of a new one wherein he appears as the new
owner of the property, he thereby in effect denied and repudiated the ownership of the other
co-owners over their shares. From this act, the statute of limitations started to run. Since an
action for conveyance.
MARIATEGUI vs. COURT of APPEALS
GR NO. 57062
JANUARY 24, 1992

FACTS: Lupo Mariategui died without a will on June 26, 1953 and contracted 3 marriages during
his lifetime. He acquired the Muntinlupa Estate while he was still a bachelor. He had 4 children
with his first wife Eusebia Montellano, who died in 1904 namely Baldomera, Maria del Rosario,
Urbano and Ireneo. Baldomera had 7 children namely Antero, Rufina, Catalino, Maria, Gerardo,
Virginia and Federico, all surnamed Espina. Ireneo on the other hand had a son named
Ruperto. On the other hand, Lupo’s second wife is Flaviana Montellano where they had a
daughter named Cresenciana. Lupo got married for the third time in 1930 with Felipa Velasco
and had 3 children namely Jacinto, Julian and Paulina. Jacinto testified that his parents got
married before a Justice of the Peace of Taguig Rizal. The spouses deported themselves as
husband and wife, and were known in the community to be such.

Lupo’s descendants by his first and second marriages executed a deed of extrajudicial partition
whereby they adjudicated themselves Lot NO. 163 of the Muntinlupa Estate and was subjected
to a voluntary registration proceedings and a decree ordering the registration of the lot was
issued. The siblings in the third marriage prayed for inclusion in the partition of the estate of
their deceased father and annulment of the deed of extrajudicial partition dated Dec. 1967.

ISSUE: Whether the marriage of Lupo with Felipa is valid in the absence of a marriage license.

RULING: Although no marriage certificate was introduced to prove Lupo and Felipa’s marriage,
no evidence was likewise offered to controvert these facts. Moreover, the mere fact that no
record of the marriage exists does not invalidate the marriage, provided all requisites for its
validity are present.

Under these circumstances, a marriage may be presumed to have taken place between Lupo
and Felipa. The laws presume that a man and a woman, deporting themselves as husband and
wife, have entered into a lawful contract of marriage; that a child born in lawful wedlock, there
being no divorce, absolute or from bed and board is legitimate; and that things have happened
according to the ordinary course of nature and the ordinary habits of life.

Hence, Felipa’s children are legitimate and therefore have successional rights.
LACBAYAN vs. SAMOY

FACTS: Betty Lacbayan (petitioner) and Bayani S. Samoy (respondent) had an illicit relationship.
During their relationship, they, together with three more incorporators, were able to establish a
manpower services company. The company acquired five parcels of land were registered in
petitioner and respondent’s names, ostensibly as husband and wife when their relationship
turned sour, they decided to divide the said properties and terminate their business
partnership by executing a Partition Agreement. Initially, respondent agreed to petitioner’s
proposal that the properties in Malvar St. and Don Enrique Heights be assigned to the latter,
while the ownership over the three other properties will go to respondent.

However, when Lacbayan wanted additional demands to be included in the partition


agreement, Samoy refused. Feeling aggrieved, petitioner filed a complaint for judicial partition
of the said properties. Petitioner’s contention: She claimed that they started to live together as
husband and wife in 1979 without the benefit of marriage and worked together as business
partners, acquiring real properties amounting to P15, 500,000.00. Respondent’s contention: He
purchased the properties using his own personal funds. RTC and CA ruled in favor or
respondent.

ISSUES: WHTHER OR NOT an action for partition precludes a settlement on the issue of
ownership.

RULING: No. While it is true that the complaint involved here is one for partition, the same is
premised on the existence or non-existence of co-ownership between the parties. Until and
unless this issue of co-ownership is definitely and finally resolved, it would be premature to
effect a partition of the disputed properties. More importantly, the complaint will not even lie if
the claimant, or petitioner in this case, does not even have any rightful interest over the subject
properties.

A careful perusal of the contents of the so-called Partition Agreement indicates that the
document involves matters which necessitate prior settlement of questions of law, basic of
which is a determination as to whether the parties have the right to freely divide among
themselves the subject properties.
CRUZ V. CATAPANG
G.R. No. 164110,
February 12, 2008

FACTS: Petitioners Leonor Cruz, Luz Cruz and Norma Maligaya are the co-owners of aparcel of
land covering an area of 1,435 square meters located at Barangay Mahabang Ludlod, Taal,
Batangas. Teofila Catapang, with the consent of Norma Maligaya as one of the aforementioned
co-owners, built a house on a lot adjacent to the subject parcel of land sometime in 1992,. The
house intruded on a portion of the co-owned property. In September 1995, Cruz visited the
property and learned about the intrusion and made several demands for Catapang to demolish
the intruding structure and vacate the portion encroaching on their property. However,
Catapang refused and disregarded her demands. Cruz then filed a complaint for forcible entry
against Catapang before the MCTC of Taal, Batangas. The MCTC decided in favor of Cruz, ruling
that consent of only one of the co-owners is not sufficient to justify defendant’s construction of
the house and possession of the portion of the lot in question. On appeal, the RTC affirmed the
decision of the MCTC in toto. Catapang filed a petition for review with the Court of Appeals,
which reversed the RTC’s decision and ruled in favor of her. The Court of Appeals held that
there is no cause of action for forcible entry in this case because respondent’s entry into the
property, considering the consent given by co-owner Norma Maligaya, cannot be characterized
as one made through strategy or stealth which gives rise to a cause of action for forcible entry.

ISSUE: Whether or not the consent given by one of the co-owners is sufficient to warrant the
dismissal of a complaint for forcible entry.

RULING: No. The consent given by one of the co-owners is not sufficient to warrant the
dismissal of a complaint for forcible entry.

Co-owners cannot devote common property to his or her exclusive use to the prejudice of the
co-ownership. A co-owner cannot give valid consent to another to build a house on the co-
owned property, which is an act tantamount to devoting the property to his or her exclusive
use. In this case, the act of Norma Maligaya is tantamount to devoting the property to her
exclusive use. Under Article 491 of the Civil Code, none of the co-owners shall, without the
consent of the others, make alterations in the thing owned in common. The Court ruled that it
would necessarily follow that none of the co-owners can, without the consent of the other co-
owners, validly give consent to the making of an alteration by another person, such as Catapang
in this case, in the thing owned in common. In addition, Article 486 of the same law states each
co-owner may use the thing owned in common provided he does so in accordance with the
purpose for which it is intended and in such a way as not to injure the interest of the co-
ownership or prevent the other co-owners from using it according to their rights. Giving
consent to a third person to construct a house on the co-owned property would be to injure the
interest of the co-ownership and would prevent other co-owners from using the property in
accordance with their rights.

In this case, the consent of only one co-owner will not warrant the dismissal of the complaint
for forcible entry filed against the respondent Catapang. The consent given by Norma Maligaya
in the absence of the consent of her other co-owners did not grant Catapang any right to enter
and even build upon the co-owned property. Respondent Catapang’s act of getting only the
consent of one co-owner, her sister Norma Maligaya, and allowing the latter to stay in the
constructed house, can in fact be considered as a strategy which she utilized in order to enter
into the co-owned property. As such, respondent’s acts constitute forcible entry. The petition
was granted.
SANTOS V. HEIRS OF LUSTRE
G.R. NO. 151016
AUGUST 6, 2008

FACTS: This petition for review seeks the reversal of the Court of Appeals (CA) Decision and
Resolution, which denied petitioners Motion to Dismiss Civil Case No. 2115, an action for
Annulment of Transfer Certificate of Title and Deed of Absolute Sale.

Dominga Lustre owned a lot which she mortgaged and later on sold to Natividad Santos, who
subsequently sold it to her son Froilan for which a TCT was issued in his name. Lustre’s heirs
Macaspac and Maniquiz filed with RTC of Gapan, Nueva Ecija a Complaint for Declaration of the
Inexistence of Contract, Annulment of Title, Reconveyance and Damages against Froilan Santos.
Lustre’s other heirs filed a Complaint for Annulment of Transfer Certificate of Title and Deed of
Absolute Sale against spouses Sofronio and Natividad Santos, Froilan Santos, Cecilia M.
Macaspac, R Transport Corporation, and the Register of Deeds of Cabanatuan City, with the
same RTC. Macaspac was impleaded as defendant in the second case because she refused to
join the other heirs as plaintiffs. Alleging that the plaintiffs’ right of action for annulment of the
Deed of Sale and TCT had long prescribed and was barred by laches, petitioners filed a Motion
to Dismiss, also on the ground of litis pendentia. The RTC denied the Motion to Dismiss. Then
they filed a petition for certiorari with the Court of Appeals (CA) which dismissed the petition
for lack of merit.

ISSUE: Whether or not the second action is barred by litis pendentia.

RULING: No. The second action is not barred by litis pendentia. It is such because there is no
identity of parties. In the case, respondents are not guilty of forum shopping because the
element of identity of parties is not present. The plaintiff in first case does not, in fact, share a
common interest with the plaintiffs in the second case. Forum shopping exists when the
elements of litis pendentia are present or when a final judgment in one case will amount to res
judicata in the other. Among its elements are identity of the parties, identity of the subject
matter and identity of the causes of action in the two cases.

Plaintiff Cecilia Macaspac in the first case filed the complaint seeking the reconveyance of the
property to her, and not to Dominga Lustre or her heirs. This is a clear act of repudiation of the
co-ownership which would negate a conclusion that she acted in privity with the other heirs or
that she filed the complaint in behalf of the co-ownership. In contrast, respondents were
evidently acting for the benefit of the co-ownership when they filed the complaint in the
second case wherein they prayed that the TCT in the name of Dominga Lustre be reinstated, or
a new certificate of title be issued in her name.
POSSESSION

POSSESSION in GOOD FAITH and BAD FAITH

PNB vs. DE JESUS


G.R. NO. 149295
SEPTEMBER 23, 2003

FACTS: Respondent filed a complaint against petitioner before the Regional Trial Court of
Occidental Mindoro for recovery of ownership and possession, with damages, over the
questioned property, 124-square-meter portion of the land covered by Transfer Certificate of
Title (TCT) No. T-17197. On 26 March 1993, he had caused a verification survey of the property
and discovered that the northern portion of the lot was being encroached upon by a building of
petitioner to the extent of 124 square meters. Despite two letters of demand sent by
respondent, petitioner failed and refused to vacate the area. Petitioner asserted that when it
acquired the lot and the building sometime in 1981 from then Mayor Bienvenido Ignacio, the
encroachment already was in existence and to remedy the situation, Mayor Ignacio offered to
sell the area in question (which then also belonged to Ignacio) to petitioner at P100.00 per
square meter which offer the latter claimed to have accepted. The sale, however, did not
materialize when, without the knowledge and consent of petitioner, Mayor Ignacio later
mortgaged the lot to the Development Bank of the Philippines. The trial court decided the case
in favor of respondent declaring him to be the rightful owner. On appeal, the Court of Appeals
sustained the trial court.

ISSUE: Whether or not petitioner is a builder in good faith.

RULING: No. Petitioner is not a builder in good faith. According to Article 448 of the Civil Code,
a builder in good faith is one who, not being the owner of the land, builds on that land believing
himself to be its owner and unaware of any defect in his title or mode of acquisition. The
essence of good faith lies in an honest belief in the validity of ones right, ignorance of a superior
claim, and absence of intention to overreach another. Applied to possession, one is considered
in good faith if he is not aware that there exists in his title or mode of acquisition any flaw
which invalidates it. Evidently, petitioner was quite aware, and indeed advised, prior to its
acquisition of the land and building from Ignacio that a part of the building sold to it stood on
the land not covered by the land conveyed to it. But petitioner was quite aware, and indeed
advised, prior to its acquisition of the land and building from Ignacio that a part of the building
sold to it stood on the land not covered by the land conveyed to it.

Equally significant is the fact that the building, constructed on the land by Ignacio, has in
actuality been part of the property transferred to petitioner. Article 448, of the Civil Code refers
to a piece of land whose ownership is claimed by two or more parties, one of whom has built
some works (or sown or planted something) and not to a case where the owner of the land is
the builder, sower, or planter who then later loses ownership of the land by sale or otherwise
for, elsewise stated, where the true owner himself is the builder of works on his own land, the
issue of good faith or bad faith is entirely irrelevant.

Petitioner is not in a valid position to invoke the aforementioned provision of the Civil Code.
PARILLA VS PILAR
G.R. NO. 167680
NOV. 30, 2006

FACTS: A tenant cannot be said to be a builder in good faith as he has no pretension to be


owner. At all events, under the Civil Code, it is the lessor who is given the option, upon
termination of the lease contract, either to appropriate the useful improvements by paying
one-half of their value at that time, or to allow the lessee to remove the improvements.
Spouses Samuel and Chinita Parilla and their son, as dealers of Pilipinas Shell Petroleum
Corporation (Pilipinas Shell), have been in possession of a parcel of land in Bantay, Ilocos Sur
which was leased to them by respondent Dr. Prospero Pilar. When the lease contract between
Pilipinas Shell and Pilar expired, and despite demands to vacate, the Parillas remained in
possession of the property on which they built improvements, the Parillas and the other
occupants remained in the property. Hence, Pilar filed a complaint for ejectment before the
Municipal Trial Court (MTC) of Bantay, Ilocos Sur. The MTC ordered the Parillas to vacate and to
pay Pilar a reasonable compensation for the use of the property. It also ordered Pilar to
reimburse the Parillas the amount Two Million Pesos representing the value of the
improvements introduced on the property. Pilar appealed to the Regional Trial Court of Vigan
and the RTC affirmed the MTC‘s Decision. However, on Pilar‘s petition for review, the Court of
Appeals set aside the lower court decision.

ISSUE: Whether or not the Parillas are entitled to reimbursement for the improvements being
builders in good faith.

RULING: Jurisprudence is replete with cases which categorically declare that Article 448 covers
only cases in which the builders, sowers or planters believe themselves to be owners of the
land or, at least, have a claim of title thereto, but not when the interest is merely that of a
holder, such as a mere tenant, agent or usufructuary. A tenant cannot be said to be a builder in
good faith as he has no pretension to be owner.

The right of the lessor upon the termination of a lease contract with respect to useful
improvements introduced on the leased property by a lessee is covered by Article 1678. Clearly,
it is Article 1678 of the New Civil Code which applies to the present case. The Parillas claim for
reimbursement of the alleged entire value of the improvements does not thus lie under Article
1678. Not even for one-half of such alleged value, there being no substantial evidence, e.g.,
receipts or other documentary evidence detailing costs of construction. Besides, by the Parillas
admission, of the structures they originally built — the billiard hall, restaurant, sari-sari store
and a parking lot, only the bodega-like sari-sari store and the parking lot now exist.

At all events, under Article 1678, it is the lessor who is given the option, upon termination of
the lease contract, either to appropriate the useful improvements by paying one-half of their
value at that time, or to allow the lessee to remove the improvements. This option solely
belongs to the lessor as the law is explicit that should the lessor refuse to reimburse said
amount, the lessee may remove the improvements, even though the principal thing may suffer
damage thereby.It appears that the lessor has opted not to reimburse.
ABALOS vs. TORIO
G.R. NO. 175444
DECEMBER 14, 2011

FACTS: On July 24, 1996, herein respondents filed a Complaint for Recovery of Possession and
Damages with the Municipal Trial Court (MTC) of Binmaley, Pangasinan against Jaime Abalos
(Jaime) and the spouses Felix and Consuelo Salazar. Respondents contended that: they are the
children and heirs of one Vicente Torio (Vicente) who died intestate on September 11, 1973; at
the time of the death of Vicente, he left behind a parcel of land measuring 2,950 square meters,
more or less, which is located at San Isidro Norte, Binmaley, Pangasinan; during the lifetime of
Vicente and through his tolerance, Jaime and the Spouses Salazar were allowed to stay and
build their respective houses on the subject parcel of land; even after the death of Vicente,
herein respondents allowed Jaime and the Spouses Salazar to remain on the disputed lot;
however, in 1985, respondents asked Jaime and the Spouses Salazar to vacate the subject lot,
but they refused to heed the demand of respondents forcing respondents to file the complaint.

Jaime and the Spouses Salazar filed their Answer with Counterclaim, denying the material
allegations in the Complaint and asserting in their Special and Affirmative Defenses that:
respondents' cause of action is barred by acquisitive prescription; the court a quo has no
jurisdiction over the nature of the action and the persons of the defendants; the absolute and
exclusive owners and possessors of the disputed lot are the deceased predecessors of
defendants; defendants and their predecessors-in-interest had been in actual, continuous and
peaceful possession of the subject lot as owners since time immemorial; defendants are
faithfully and religiously paying real property taxes on the disputed lot as evidenced by Real
Property Tax Receipts; they have continuously introduced improvements on the said land, such
as houses, trees and other kinds of ornamental plants which are in existence up to the time of
the filing of their Answer.

ISSUE: Whether the petitioners and their predecessors-in-interest possessed the disputed lot in
the concept of an owner, or whether their possession is by mere tolerance of respondents and
their predecessors-in-interest.

RULING: The Court agrees with the observation of respondents that some of the petitioners in
the instant petition were the intervenors when the case was filed with the MTC. Records would
show that they did not appeal the Decision of the MTC. The settled rule is that failure to perfect
an appeal renders the judgment final and executory. Hence, insofar as the intervenors in the
MTC are concerned, the judgment of the MTC had already become final and executory.

Petitioners claim that they have acquired ownership over the disputed lot through ordinary
acquisitive prescription. Acquisitive prescription of dominion and other real rights may be
ordinary or extraordinary. Ordinary acquisitive prescription requires possession in good faith
and with just title for ten (10) years. Without good faith and just title, acquisitive prescription
can only be extraordinary in character which requires uninterrupted adverse possession for
thirty (30) years.

Possession in good faith consists in the reasonable belief that the person from whom the thing
is received has been the owner thereof, and could transmit his ownership. There is just title
when the adverse claimant came into possession of the property through one of the modes
recognized by law for the acquisition of ownership or other real rights, but the grantor was not
the owner or could not transmit any right. The disputed land was shown to have already been
validly sold to Marcos Torio, who, thereupon, assigned the same to his son Vicente, the father
of petitioners [herein respondents]. A valid sale was amply established and the said validity
subsists because the deed evidencing the same was duly notarized.
ACQUISITION of POSSESSION

BUNYI VS. FACTOR


G.R. NO. 172547
JUNE 30, 2009

FACTS: Fe S. Factor is one of the co-owners of a piece of land. The ownership of the land
originated from respondents paternal grandparents Constantino Factor and Maura Mayuga-
Factor who had been in actual, continuous, peaceful, public, adverse and exclusive possession
and occupation of the land even before 1906.

On December 9, 1975, the children of Constantino Factor and Maura Mayuga-Factor filed a
Petition for Original Registration and Confirmation of Imperfect Title to the said parcel of land.
The trial court granted the petition and declared the children of Constantino Factor and Maura
Mayuga-Factor as co-owners of the property. The children thereafter sold part of the Factor
family property during the same year. The siblings, except Enrique Factor, respondent’s father,
shared and divided the proceeds of the sale among themselves, with the agreement that
Enrique would have as his share the portion of the property located in Antioch Street, Pilar
Executive Village, Almanza I, Las Pias City, known as the Factor compound.

Enrique caused the construction of several houses in the compound including the subject
property, a rest house, where members of the Factor family stayed during get-togethers and
visits. Petitioners Precy Bunyi and her mother, Mila Bunyi, were tenants in one of the houses
inside the compound. When Enrique Factor died on August 7, 1993, the administration of the
Factor compound including the subject rest house and other residential houses for lease was
transferred and entrusted to Enriques eldest child, Gloria Factor-Labao.

Gloria Factor-Labao, together with her husband Ruben Labao and their son Reggie F. Labao,
lived in Tipaz, Taguig, Metro Manila but visited and sometimes stayed in the rest house because
Gloria collected the rentals of the residential houses and oversaw the Factor compound. When
Gloria died on January 15, 2001, the administration and management of the Factor compound
including the subject rest house, passed on to respondent Fe S. Factor as co-owner of the
property. As an act of goodwill and compassion, considering that Ruben Labao was sickly and
had no means of income, respondent allowed him to stay at the rest house for brief, transient
and intermittent visits as a guest of the Factor family.

On May 31, 2002, Ruben Labao married petitioner Precy Bunyi. On November 10, 2002, Ruben
Labao died. At about this time, respondent discovered that petitioners forcibly opened the
doors of the rest house and stole all the personal properties owned by the Factor family and
then audaciously occupied the premises. Respondent alleged that petitioners unlawfully
deprived her and the Factor family of the subject propertys lawful use and possession.
Petitioners, for their part, questioned Fes claim of ownership of the subject property and the
alleged prior ownership of her father Enrique Factor. They asserted that the subject property
was owned by Ruben Labao, and that petitioner Precy with her husband moved into the subject
property.

On July 13, 2004, the Metropolitan Trial Court (MeTC) of Las Pias City, Branch 79 ruled in favor
of Fe S. Factor. Petitioners appealed the decision to the RTC of Las Pias City, Branch 198, which,
however, affirmed in toto the decision of the MeTC and later denied their motion for
reconsideration. Undaunted, petitioners filed a petition for review before the Court of Appeals
but it was denied also.

ISSUE: Who has the better right of physical and material possession of the subject property?
RULING: The respondent. In ejectment cases, the only issue for resolution is who is entitled to
the physical or material possession of the property involved, independent of any claim of
ownership set forth by any of the party-litigants. The one who can prove prior possession de
facto may recover such possession even from the owner himself. Possession de facto is the
physical possession of real property. Possession de facto and not possession de jure is the only
issue in a forcible entry case. This rule holds true regardless of the character of a partys
possession, provided, that he has in his favor priority of time which entitles him to stay on the
property until he is lawfully ejected by a person having a better right by either accion publiciana
or accion reivindicatoria.

For one to be considered in possession, one need not have actual or physical occupation of
every square inch of the property at all times. Possession can be acquired not only by material
occupation, but also by the fact that a thing is subject to the action of ones will or by the proper
acts and legal formalities established for acquiring such right. Possession can be acquired by
juridical acts. These are acts to which the law gives the force of acts of possession. Examples of
these are donations, succession, execution and registration of public instruments, and the
inscription of possessory information titles.

While petitioners claim that respondent never physically occupied the subject property, they
failed to prove that they had prior possession of the subject property. On the other hand, it was
established that respondents grandparents, Constantino Factor and Maura Mayuga-Factor, had
been the occupants and in possession of various agricultural parcel of lands situated in
Almanza, Las Pias City, in the concept of owners, for more than thirty years prior to 1975.
The right of respondents predecessors over the subject property is more than sufficient to
uphold respondents right to possession over the same. Respondent’s right to the property was
vested in her along with her siblings from the moment of their fathers death. As heir,
respondent had the right to the possession of the property, which is one of the attributes of
ownership. Such rights are enforced and protected from encroachments made or attempted
before the judicial declaration since respondent acquired hereditary rights even before judicial
declaration in testate or intestate proceedings.
After the death of Enrique Factor, it was his eldest child, Gloria Factor-Labao who took over the
administration of the subject property. And as a consequence of co-ownership, soon after the
death of Gloria, respondent, as one of the surviving co-owners, may be subrogated to the rights
of the deceased co-owner, which includes the right to the administration and management of
the subject property.
PEOPLE vs. PENAFLORIDA
G.R. NO. 175604
APRIL 10, 2008

FACTS: SPO3 Vicente Competentereceived a tip from an asset that a bundle of marijuana was
being transported by appellant to Huyon-huyon from another barangay in Tigaon, Camarines
Sur. (Agravante), chief of police of Tigaon, then organized a team composed of Competente as
team leader, SPO2 Callo, SPO1 Portugal, PO3 Pillos and PO2 Edgar Latam. The team boarded
the police mobile car and proceeded to Barangay Huyon-huyon.They overtook appellant who
was on a bicycle. The police officers flagged appellant down and found marijuana wrapped in a
cellophane and newspaper together with other grocery items. The amount of P1550.00 was
also found in appellant's possession. The police officers confiscated these items and took
photographs thereof. Appellant was then brought to the headquarters where he was booked.

Major Lorlie Arroyo, a forensic chemist at the PNP Crime Laboratory Regional Office No. V, was
presented as an expert witness to identify the subject marijuana leaves. She related that after
taking a representative sample from the 928-gram confiscated dried leaves, the same was
tested positive of marijuana.

Appellant denied the accusations against him.That on his way home, they met Boyet Obias who
requested appellant to bring a package wrapped in a newspaper to Jimmy Gonzales, he placed
it in the basket in front of his bicycle and Gonzales proceeded to the Tiagon town proper. On his
way home, he was flagged down by the police and was invited to go with them to the
headquarters.

ISSUE: Whether or not the contention of the appellant is tenable?

RULING: No. The police was tipped off at around 1:00 p.m. that appellant was transporting
marijuana to Huyon-huyon. Certainly, they had no time to secure an arrest warrant as appellant
was already in transit and already committing a crime. The arrest was effected after appellant
was caught in flagrante delicto. He was seen riding his bicycle and carrying with him the
contraband, hence, demonstrating that a crime was then already being committed. Under the
circumstances, the police had probable cause to believe that appellant was committing a crime.
Thus, the warrantless arrest is justified.
SOLEDAD vs. PEOPLE OF THE PHILIPPINES
G.R. No. 184274
February 23, 2011

FACTS: Sometime in June 2004, private complainant Henry C. Yu received a call on his mobile
phone from a certain Tess or Juliet Villar, a credit card agent, who offered a Citifinancing loan
assistance at a low interest rate. Enticed by the offer, private complainant invited Rochelle
Bagaporo to go to his office in Quezon City. While in his office, Rochelle Bagaporo indorsed
private complainant to her immediate boss, a certain Arthur [later identified as petitioner]. In
their telephone conversation, told private complainant to submit documents to a certain Carlo
(later identified as Ronald Gobenchiong). Private complainant submitted various documents,
such as his Globe handyphone original platinum gold card, identification cards and statements
of accounts. Subsequently, private complainant followed up his loan status but he failed to get
in touch with either [petitioner] or Ronald Gobenchiong.

During the first week of August 2004, private complainant received his Globe handyphone
statement of account wherein he was charged for two (2) mobile phone numbers which were
not his. Upon verification with the phone company, private complainant learned that he had
additional five (5) mobile numbers in his name, and the application for said cellular phone lines
bore the picture of [petitioner] and his forged signature. Private complainant also checked with
credit card companies and learned that his Citibank Credit Card database information was
altered and he had a credit card application with Metrobank Card Corporation (Metrobank).

Thereafter, private complainant and Metrobanks junior assistant manager Jefferson Devilleres
lodged a complaint with the National Bureau of Investigation (NBI) which conducted an
entrapment operation.

During the entrapment operation, NBIs Special Investigator (SI) Salvador Arteche [Arteche],
together with some other NBI operatives, arrived in Las Pias around 5:00 P.M. [Arteche] posed
as the delivery boy of the Metrobank credit card. Upon reaching the address written on the
delivery receipt, [Arteche] asked for Henry Yu. [Petitioner] responded that he was Henry Yu and
presented to [Arteche] two (2) identification cards which bore the name and signature of
private complainant, while the picture showed the face of [petitioner]. [Petitioner] signed the
delivery receipt. Thereupon, [Arteche] introduced himself as an NBI operative and
apprehended [petitioner]. [Arteche] recovered from [petitioner] the two (2) identification cards
he presented to [Arteche] earlier.

ISSUE: Whether or not Soledad is guilty of violation of Section 9, R.A. No. 8484 for possessing a
counterfeit access device or access device fraudulently applied for.

RULING: Observing the foregoing guidelines, the Court deems it proper to impose upon the
petitioner the indeterminate sentence of six (6) months and one (1) day of prision correccional
as minimum to six (6) years, eight (8) months and twenty-one (21) days of prision mayor as
maximum.
CHUA-BURCE VS. COURT OF APPEALS
G.R. NO. 109595
APRIL 27, 2000

FACTS: Ramon Rocamora, manager of Metrobank, requested FructuosoPenaflor, Assistant


Cashier, to conduct a physical bundle count of cash inside the vault, which should total to P4
million. They found out that there was a shortage of P150, 000. After 4 investigations
conducted by the bank and NBI, the reports concluded that Cristeta Chua-Burce, Cash
Custodian, was primary responsible for the shortage. Unable to explain the shortage, the
services of the accused was terminated.

Chua-Burce, together with her husband Antonio Burce, were charged with the crime of estafa.
A civil case was also instituted. The accused prayed for suspension of criminal case due to a
prejudicial question. It was first granted but denied by the CA. The CRIMINAL and CIVIL cases
continued.

ISSUE: Whether or not the elements of estafa were proven beyond reasonable doubt.

RULING: No, the crime of estafa was not proven.

The elements of Estafa, ART. 315 (1) (b), are the following:

a) The personal property is received in trust, on commission, for administration, or any


other circumstances, with the duty return.

b) There is a conversion/diversion of such property or denial that he received it.

c) Such conversion/diversion is to the injury of another.

d) There is demand for such property

The 1st element is absent. The 1st element gives the tranferee both material and juridical
possession of the personal property. Juridical possession means the transferee has a right over
the thing which he may even set up against the owner. The possession of the accused of the
money had no juridical possession. Being a cash custodian, her possession is akin to that of a
bank teller. And possession of a bank teller is possession of the bank. She was a mere
custodian. She should have been charged with qualified theft, but double jeopardy is already in
play. Difference between an agent and teller. TELLER – payment to the teller is a payment to
the bank, he is a mere custodian. AGENT – he can assert his independent, autonomous right to
retain money, even against the owner.
EFFECTS of POSSESSION

EDCA PUBLISHING AND DISTRIBUTING CORP. vs. SANTOS


184 SCRA 614

FACTS:
On a relevant date, one person who identified himself as Professor Jose Cruz placed an order
through telephone with Edca Publishing. He ordered 400+ books and issued a personal check as
payment. Then he sold some of the books to Santos who, after verifying the seller’s ownership
from the invoice shown, paid Cruz. Meanwhile, Edca being suspicious over the second order
placed by Cruz verified with De La Salle College where he had claimed to be dean and was
informed that no such person was under its employ. It was also found out that there was no
account with the bank against which he had drawn his check. It was later found out that his real
name was Tomas de la Pena. Edca reported this to the police and through an entrapment, de la
Pena was captured. On the same date, Edca sought the assistance of the police in recovery of
the books bought from it. They forced their way inside Santos’ store and seized the books
without any warrant.

ISSUE: Whether or not the owner was unlawfully deprived of the property?

RULING: No. Santos was a good faith buyer after taking steps to verify the identity of the seller.
When she was showed the invoice, she reasonably believed that he was a legitimate seller.
With regard to unlawful deprivation, EDCA was not unlawfully deprived of the property by
mere failure of consideration. There was already a perfected contract of sale. Proof was even
substantiated when EDCA gave the invoice as proof of payment upon delivery of the books. This
did not amount to unlawful taking, because by the delivery of EDCA to Cruz, ownership of the
books already transferred to him.

First, the contention of petitioner that Santos has not established ownership over the disputed
books because they have not even shown the receipt evidencing the purchase is without merit.
The possession of movable property acquired in good faith is equivalent to title. Second, Santos
acquired the books in good faith as found by the lower courts. She first ascertained the
ownership and relied on the invoice shown to her by de la Pena. Santos was in the business of
buying and selling books and often deal with hard-up sellers who urgently have to part with
their books at reduced prices. Third, and on the real issue, on whether Edca had been
unlawfully deprived of the books, Edca argued that the impostor acquired no title to the books
because of the lack of funds in the check issued and want of consideration. This is without
merit. Non-payment of purchase price only gives rise to the right to demand payment or
rescission of the contract. Actual delivery was made to the impostor and thus, ownership was
acquired by him. Non-payment was a matter privy to him and Edca and doesn't involve Santos
who later acquired the books.
BPI FAMILY BANK vs. AMADO FRANCO
G.R. No. 123498
November 23, 2007

FACTS: On August 15, 1989, Tevesteco Arrastre-Stevedoring Co., Inc. (Tevesteco) opened a
savings and current account with BPI-FB. Soon thereafter, or on August 25, 1989, First Metro
Investment Corporation (FMIC) also opened a time deposit account with the same branch of
BPI-FB with a deposit of P100,000,000.00, to mature one year thence.

Subsequently, on August 31, 1989, Franco opened three accounts, namely, a current, savings,
and time deposit with BPI-FB. The current and savings accounts were respectively funded with
an initial deposit of P500,000.00 each, while the time deposit account had P1,000,000.00 with a
maturity date of August 31, 1990. The total amount of P2, 000, 000.00 used to open these
accounts is traceable to a check issued by Tevesteco allegedly in consideration of Francos
introduction of Eladio Teves, who was looking for a conduit bank to facilitate Tevestecos
business transactions, to Jaime Sebastian, who was then BPI-FB SFDMs Branch Manager. In
turn, the funding for the P2,000,000.00 check was part of the P80,000,000.00 debited by BPI-FB
from FMICs time deposit account and credited to Tevestecos current account pursuant to an
Authority to Debit purportedly signed by FMICs officers.

It appears, however, that the signatures of FMICs officers on the Authority to Debit were
forged. On September 4, 1989, Antonio Ong, upon being shown the Authority to Debit,
personally declared his signature therein to be a forgery. Unfortunately, Tevesteco had already
effected several withdrawals from its current account (to which had been credited the
P80,000,000.00 covered by the forged Authority to Debit) amounting to P37,455,410.54,
including the P2,000,000.00 paid to Franco.

ISSUE: Whether or not the respondent had better right to the deposits in the subject accounts
which are part of the proceeds of a forged Authority to Debit.

RULING: No. There is no doubt that BPI-FB owns the deposited monies in the accounts of
Franco, but not as a legal consequence of its unauthorized transfer of FMICs deposits to
Tevestecos account. BPI-FB conveniently forgets that the deposit of money in banks is governed
by the Civil Code provisions on simple loan or mutum. As there is a debtor-creditor relationship
between a bank and its depositor, BPI-FB ultimately acquired ownership of Francos deposits,
but such ownership is coupled with a corresponding obligation to pay him an equal amount on
demand. Although BPI-FB owns the deposits in Francos accounts, it cannot prevent him from
demanding payment of BPI-FBs obligation by drawing checks against his current account, or
asking for the release of the funds in his savings account. Thus, when Franco issued checks
drawn against his current account, he had every right as creditor to expect that those checks
would be honored by BPI-FB as debtor.

More importantly, BPI-FB does not have a unilateral right to freeze the accounts of Franco
based on its mere suspicion that the funds therein were proceeds of the multi-million peso
scam Franco was allegedly involved in. To grant BPI-FB, or any bank for that matter, the right to
take whatever action it pleases on deposits which it supposes are derived from shady
transactions, would open the floodgates of public distrust in the banking industry.
EASEMENTS OR SERVITUDE

Restrictive Covenant

FAJARDO VS. FREEDOM TO BUILD


G.R. NO. 134692
AUGUST 1, 2000

FACTS: Freedom to Build Inc., an owner-developer and seller of low-cost housing sold to
petitioner-spouses a house and lot. The Contract to sell executed between the parties,
contained a Restrictive Covenant providing certain prohibitions, to wit:

“Easements. For the good of the entire community, the homeowner must observe a two-meter
easement in front. No structure of any kind (store, garage, bodega, etc.) may be built on the
front easement. “Upward expansion. A second storey is not prohibited. But the second storey
expansion must be placed above the back portion of the house and should not extend forward
beyond the apex of the original building. “Front expansion: 2nd Storey: No unit may be
extended in the front beyond the line as designed and implemented by the developer in the 60
sq. m. unit. In other words, the 2nd floor expansion, in front, is 6 meters back from the front
property line and 4 meters back from the front wall of the house, just as provided in the 60 sq.
m. units.” The above restrictions were also contained in Transfer Certificate of Title No. N-
115384 covering the lot issued in the name of petitioner-spouses.

The controversy arose when the petitioners despite repeated demand from the respondent,
extended the roof of their house to the property line and expanded the second floor of their
house to a point directly above the original front wall.

ISSUE: Whether or not the provisions of the Restrictive Covenant are valid.

RULING: Yes. Petitioners must be held to be bound thereby. Since the extension constructed
exceeds the floor area limits of the Restrictive Covenant, petitioner-spouses can be required to
demolish the structure to the extent that it exceeds the prescribed floor area limits.

Restrictive covenants are not, strictly speaking, synonymous with easements. While it may be
correct to state that restrictive covenants on the use of land or the location or character of
buildings or other structures thereon may broadly be said to create easements or rights, it can
also be contended that such covenants, being limitations on the manner in which one may use
his own property, do not result in true easements,[7] but a case of servitudes (burden),
sometimes characterized to be negative easements or reciprocal negative easements. Negative
easement is the most common easement created by covenant or agreement whose effect is to
preclude the owner of the land from doing an act, which, if no easement existed, he would be
entitled to do.

The provisions in a restrictive covenant prescribing the type of the building to be erected are
crafted not solely for the purpose of creating easements, generally of light and view, nor as a
restriction as to the type of construction, but may also be aimed as a check on the subsequent
uses of the building conformably with what the developer originally might have intended the
stipulations to be.
ABELLANA vs. COURT OF APPEALS
G.R. No. 97039
April 24, 1992

FACTS: Petitioners are owners of a parcel of land on the northwest side of Nonoc Subdivision,
Cebu. They sued to establish an easement of a right of way over a subdivision road, which they
claim they have acquired through prescription since their ancestors have been using these since
time immemorial. They pray that the concrete wall surrounding the village be taken down to
allow easy access to the public highway.

RTC held in favor of the petitioners ordering the respondents to demolish the subject fences or
enclosures at the dead ends of the subject lot at their expense and to leave them open for the
use of the plaintiffs and the general public.

However, the Court of Appeals reversed the lower court’s decision averring that road lots in
subdivisions are private property and may only be used as a public highway once acquired by
the government through donation, purchase or expropriation. The petitioner filed for a motion
for reconsideration which was also denied. Hence, the petition of the instant case.

ISSUE: Whether or not the easement of a right of way may be acquired by prescription?

RULING: No. Article 620 of the Civil Code provides that only continuous and apparent
easements may be acquired by prescription. The easement of a right of way cannot be
considered continuous because its use is at intervals and is dependent on the acts of man.
WHEREFORE, finding no merit in the petition for review, the same is DENIED with costs against
the petitioners.
BICOL AGRO-INDUSTRIAL PRODUCERS COOPERATIVE, INC. vs. EDMUNDO O. OBIAS
G.R. No. 172077
October 9, 2009

FACTS: BISUDECO constructed a road measuring approximately 7 meters wide and 2.9
kilometers long. The disputed road was used by BISUDECO in hauling and transporting
sugarcane to and from its mill site and has thus become indispensable to its sugar milling
operations.

Respondents unjustifiably barricaded the disputed road, preventing petitioners and the other
sugar planter’s vehicles from passing through the disputed road, thereby causing serious
damage and prejudice to petitioner. Petitioner alleged that BISUDECO constructed the disputed
road pursuant to an agreement with the owners of the ricefields the road traversed. The
agreement provides that BISUDECO shall employ the children and relatives of the landowners
in exchange for the construction of the road on their properties. Petitioner contends that
through prolonged and continuous use of the disputed road, BISUDECO acquired a right of way
over the properties of the landowners, which right of way in turn was acquired by it when it
bought BISUDECOs assets.
Respondents denied having entered into an agreement with BISUDECO regarding the
construction and the use of the disputed road. They alleged that BISUDECO, surreptitiously and
without their knowledge and consent, constructed the disputed road on their properties and
has since then intermittently and discontinuously used the disputed road for hauling sugarcane
despite their repeated protests. Respondents claimed that the road has become a public road,
since no public funds were used for its construction and maintenance.

ISSUES: Whether or not there is an existing agreement between BISUDECO and Respondents.

RULING: NO. Easement or servitude is an encumbrance imposed upon an immovable for the
benefit of another immovable belonging to a different owner. By its creation, easement is
established either by law (in which case it is a legal easement) or by will of the parties (a
voluntary easement). In terms of use, easement may either be continuous or discontinuous.
The easement of right of way the privilege of persons or a particular class of persons to pass
over anothers land, usually through one particular path or linen is characterized as a
discontinuous easement because its use is in intervals and depends on the act of man. Because
of this character, an easement of a right of way may only be acquired by virtue of a title.
Art. 622. Continuous non-apparent easements, and discontinuous ones, whether apparent or
not, may be acquired only by virtue of a title.
Based on the foregoing, in order for petitioner to acquire the disputed road as an easement of
right-of-way, it was incumbent upon petitioner to show its right by title or by an agreement
with the owners of the lands that said road traversed. While conceding that they have no direct
evidence of the alleged agreement, petitioner posits that they presented circumstantial
evidence which, if taken collectively, would prove its existence. Specifically, petitioner cites the
following circumstances, to wit:

a. The agreement was of public knowledge. Allegedly BISUDECO and respondents entered into
an agreement for the construction of the road provided that the latter, their children or
relatives were employed with BISUDECO.
b. The road was continuously used by BISUDECO and the public in general.
c. There was no protest or complaint from respondents for almost a period of two decades.
d. The portions of the land formerly belonging to respondents affected by the road were
already segregated and surveyed from the main lots.
e. The road in dispute is already a barangay road.
QUINTANILLA VS. ABANGAN
G.R. No. 160613
February 12, 2008

FACTS: Sometime in the 1960s, Perfecta bought Lot No. 3771-B-1-A, with an area of 2,244
square meters, located at Inayawan, Cebu City (the dominant estate) from one Dionisio
Abasolo, who formerly owned all the properties therein. Thereafter, Perfecta donated the
dominant estate to Apolinardito, who is now the registered owner thereof. Petitioners own QC
Rattan Inc., a domestic corporation engaged in the manufacture and export of rattan-made
furniture. In the conduct of their business, they use vans to haul and transport raw materials
and finished products. As they wanted to expand their business and construct a warehouse on
their property (the dominant estate), they asked for a right of way from Pedro sometime in
April 1994.

However, it appears that Pedro, who was the owner of Lot No. 3771-A-1, containing an area of
1,164 square meters (the servient estate) and a lot near the dominant estate, sold the same to
DARYL'S on March 24, 1994, and thereafter, DARYL'S constructed a warehouse over the
servient estate, enclosing the same with a concrete fence.

Petitioners, thus, sought the imposition of an easement of right of way, six (6) meters in width,
or a total area of 244 square meters, over the servient estate.

ISSUE: Whether or not the claim of the easement of right of way is proper.

RULING: As provided for under the provisions of Article 650 of the New Civil Code, the
easement of right of way shall be established at the point least prejudicial to the servient
estate, and, insofar as consistent with this rule, where the distance from the dominant estate to
a public highway may be the shortest. Where there are several tenements surrounding the
dominant estate, and the easement may be established on any of them, the one where the way
is shortest and will cause the least damage should be chosen. But if these two circumstances do
not concur in a single tenement, as in the instant case, the way which will cause the least
damage should be used, even if it will not be the shortest. The criterion of least prejudice to the
servient estate must prevail over the criterion of shortest distance. The court is not bound to
establish what is the shortest; a longer way may be established to avoid injury to the servient
tenement, such as when there are constructions or walls which can be avoided by a round-
about way, as in the case at bar.

As between a right of way that would demolish a fence of strong materials to provide ingress
and egress to a public highway and another right of way which although longer will only require
a van or vehicle to make a turn, the second alternative should be preferred. Mere convenience
for the dominant estate is not what is required by law as the basis for setting up a compulsory
easement. Even in the face of necessity, if it can be satisfied without imposing the easement,
the same should not be imposed.
QUIMEN vs. COURT OF APPEALS
G.R. No. 112331
May 29, 1996

FACTS: In easement of right of way that easement where the way is shortest and will cause
least prejudice shall be chosen. However, if the two circumstances do not concur in a single
tenement, the way where damage will be least shall be used even if not the shortest route.[1]
This is so because least prejudice prevails over shortest distance. This means that the court is
not bound to establish what is the shortest distance; a longer way may be adopted to avoid
injury to the servient estate, such as when there are constructions or walls which can be
avoided by a roundabout way, or to secure the interest of the dominant owner, such as when
the shortest distance would place the way on a dangerous decline.

Thus we conclude from the succeeding facts: Petitioner Anastacia Quimen together with her
brothers and sister Rufina inherited a piece of property. They agreed to subdivide the property
equally among themselves, as they did, with the shares of Anastacia, Sotero, Sulpicio and
Rufina abutting the municipal road. The share of Anastacia, located at the extreme left, was
designated as Lot No. 1448-B- 1. It is bounded on the right by the property of Sotero designated
as Lot. No. 1448-B-2. Adjoining Soteros property on the right are Lots Nos. 1448-B-3 and 1448-
B-4 originally owned by Rufina and Sulpicio, respectively, but which were later acquired by a
certain Catalina Santos. Located directly behind the lots of Anastacia and Sotero is the share of
their brother Antonio designated as Lot No. 1448-B-C which the latter divided into two (2)
equal parts, now Lots Nos. 1448-B-6-A and 1448-B-6-B, each with an area of 92 square meters.
Lot No. 1448-B-6-A is located behind Anastacias Lot No. 1448-B-1, while Lot No. 1448-B-6-B is
behind the property of Sotero, father of respondent Yolanda.

ISSUE: Whether or not Yolanda was entitled to a right of way.

RULING: The trial court found that Yolanda’s property was situated at the back of her father’s
property and held that there existed an available space of about nineteen (19) meters long
which could conveniently serve as a right of way between the boundary line and the house of
Yolanda s father; that the vacant space ended at the left back of Soteros store which was made
of strong materials; that this explained why Yolanda requested a detour to the lot of Anastacia
and cut an opening of one (1) meter wide and five (5) meters long to serve as her right of way
to the public highway. But notwithstanding its factual observations, the trial court concluded,
although erroneously, that Yolanda was not entitled to a right of way on petitioner’s property
since a detour through it would not make the line straight and would not be the route shortest
to the public highway.

In applying Art. 650 of the New Civil Code, respondent Court of Appeals declared that the
proposed right of way of Yolanda, which is one (1) meter wide and five (5) meters long at the
extreme right of petitioners property, will cause the least prejudice and/or damage as
compared to the suggested passage through the property of Yolanda s father which would
mean destroying the sari-sari store made of strong materials. Absent any showing that these
findings and conclusion are devoid of factual support in the records, or are so glaringly
erroneous, this Court accepts and adopts them. As between a right of way that would demolish
a store of strong materials to provide egress to a public highway, and another right of way
which although longer will only require an avocado tree to be cut down, the second alternative
should be preferred. After all, it is not the main function of this Court to analyze or weigh the
evidence presented all over again where the petition would necessarily invite calibration of the
whole evidence considering primarily the credibility of witnesses, existence and relevancy of
specific surrounding circumstances, their relation to each other, and the probabilities of the
situation.
UNISOURCE COMMERCIAL AND DEVELOPMENT CORPORATION VS. CHUNG
G.R. No. 173252
July 17, 2009

FACTS: Unisource Commercial and Development Corporation is the registered owner of a


parcel land of which contains a memorandum of encumbrance of a voluntary easement carried
over from the OCT of Encarnacion S. Sandico declaring that Francisco Hidalgo has the right to
open doors in the course of his lot and to pass through the land of Encarnacion Sandico, until
the bank of the estero that goes to the Pasig River, and towards the right of a Callejon. The
annotation does not expressly provide that it will be binding to the heirs and assigns of the
parties. Furthermore, the property of Hidalgo was already subdivided.

The memorandum of encumbrance was consistently annotated at the back of every title
covering Sandico’s property until it was acquired by the petitioner. Hidalgo’s property, on the
other hand was eventually transferred to respondents Joseph, Kia and Cleto all surnamed
Chung.

Petitioner filed a Petition to Cancel the Encumbrance of Voluntary Easement of Right of Way on
the ground that the dominant estate has an adequate access to a public road which is Matienza
Street which was granted by the trial court but eventually reversed by the Court of Appeals.

ISSUES: Whether or not the Encumbrance of Voluntary Easement of Right of Way can be
cancelled by the petitioners who own the servient estate on the ground that that the dominant
estate has an adequate access to a public road.

RULING:

An easement is a real right on another’s property, corporeal and immovable, whereby the
owner of the latter must refrain from doing or allowing somebody else to do or something to
be done on his property, for the benefit of another person or tenement. These are established
either by law (legal easement) or by the will of the owner (voluntary easement).

The opening of an adequate outlet to a highway can extinguish only legal or compulsory
easements, not voluntary easements like in the case at bar. Petitioner itself admitted that the
existing easement is voluntary and cannot now claim that what exists is a legal easement and
that the same should be cancelled since the dominant estate is not an enclosed estate as it has
an adequate access to a public road which is Callejon Matienza Street. The fact that an
easement by grant may have also qualified as an easement of necessity does not detract from
its permanency as a property right, which survives the termination of the necessity. This
easement of right of way, like any other contract, could be extinguished only by mutual
agreement or by renunciation of the owner of the dominant estate. (Art. 631,NCC)
NUISANCE

CLASSIFICATION of NUISANCE and REMEDIES

TELMO vs. BUSTAMANTE


G.R. NO. 182567
July 13, 2009

FACTS: The complaint alleged that respondent is a co-owner of a real property, known as Lot
952-A. Petitioner and Elizalde Telmoare the owners of the two (2) parcels of land denominated
as Lot 952-B and 952-C, respectively, located at the back of respondents lot. When his lot was
transgressed by the construction of the Noveleta-Naic-Tagaytay Road, respondent offered for
sale the remaining lot to the Telmos. The latter refused because they said they would have no
use for it, the remaining portion being covered by the roads 10-meter easement.

The complaint further alleged that, on May 8, 2005, respondent caused the resurvey of Lot 952-
A in the presence of the Telmos. The resurvey showed that the Telmos encroached upon
respondent’s lot. Petitioner then uttered, “Hanggat ako ang municipal engineer ng Naic, Cavite,
hindi kayo makakapagtayo ng anuman sa lupa nyo; hindi ko kayo bibigyan ng building permit”.
On May 10, 2005, respondent put up concrete poles on his lot. However, that same day, the
Telmos and their men allegedly destroyed the concrete poles. The following day, respondent’s
relatives went to Brgy. Chairman Consumo to report the destruction of the concrete poles.
Consumo told them that he would not record the same, because he was present when the
incident occurred.

Respondent complained that he and his co-owners did not receive any just compensation from
the government when it took a portion of their property for the construction of the Noveleta-
Naic- Tagaytay Road. Worse, they could not enjoy the use of the remaining part of their lot due
to the abusive, Illegal, and unjust acts of the Telmos and Consumo.

ISSUE: Whether or not the property claimed and enclosed with concrete posts by respondent
was validly taken by the National Government through its power of eminent domain.

RULING: Petitioner contends that respondent’s concrete posts were in the nature of a nuisance
per se, which may be the subject of summary abatement sans any judicial proceedings. Again,
we disagree.

A nuisance per se is that which affects the immediate safety of persons and property and may
be summarily abated under the undefined law of necessity. Evidently, the concrete posts
summarily removed by petitioner did not at all pose a hazard to the safety of persons and
properties, which would have necessitated immediate and summary abatement. What they did,
at most, was to pose an inconvenience to the public by blocking the free passage of people to
and from the national road.
GANCAYCO VS. QUEZON CITY
G.R. No. 177807
October 11, 2011

FACTS: Retired Justice Emilio A. Gancayco bought a parcel of land located EDSA, Quezon City. A
few years later, the Quezon City Council issued Ordinance No. 2904, entitled "An Ordinance
Requiring the Construction of Arcades, for Commercial Buildings to be Constructed in Zones
Designated as Business Zones in the Zoning Plan of Quezon City, and Providing Penalties in
Violation Thereof.

It required the relevant property owner to construct an arcade along EDSA. An arcade is
defined as any portion of a building above the first floor projecting over the sidewalk beyond
the first storey wall used as protection for pedestrians against rain or sun.

It bears emphasis that at the time Ordinance No. 2904 was passed by the city council, there was
yet no building code passed by the national legislature. Thus, the regulation of the construction
of buildings was left to the discretion of local government units. Under this particular
ordinance, the city council required that the arcade is to be created by constructing the wall of
the ground floor facing the sidewalk a few meters away from the property line. Thus, the
building owner is not allowed to construct his wall up to the edge of the property line, thereby
creating a space or shelter under the first floor. The ordinance covered the property of Justice
Gancayco.

The MMDA sent a notice of demolition to Justice Gancayco alleging that a portion of his
building violated the National Building Code of the Philippines in relation to Ordinance No.
2904. He did not comply with the notice. The MMDA then proceeded to demolish the party wall
of the ground floor structure. The City Government of Quezon City claimed that the ordinance
was a valid exercise of police power, regulating the use of property in a business zone.

ISSUE: Whether Ordinance No. 2094 is a valid exercise of police power.

RULING: Yes, it is a valid delegation of Police Power. Police power is an inherent attribute of
sovereignty. It has been defined as the power vested by the Constitution in the legislature to
make, ordain, and establish all manner of wholesome and reasonable laws, statutes and
ordinances, either with penalties or without, not repugnant to the Constitution, as they shall
judge to be for the good and welfare of the commonwealth, and for the subjects of the same.
The power is plenary and its scope is vast and pervasive, reaching and justifying measures for
public health, public safety, public morals, and the general welfare. In the exercise of police
power, property rights of individuals may be subjected to restraints and burdens in order to
fulfill the objectives of the government. For this reason, when the conditions so demand as
determined by the legislature, property rights must bow to the primacy of police power
because property rights, though sheltered by due process, must yield to general welfare. Police
power as an attribute to promote the common good would be diluted considerably if on the
mere plea of petitioners that they will suffer loss of earnings and capital, the questioned
provision is invalidated. Moreover, in the absence of evidence demonstrating the alleged
confiscatory effect of the provision in question, there is no basis for its nullification in view of
the presumption of validity, which every law has in its favor. It is clear that the primary
objectives of the city council of Quezon City when it issued the questioned ordinance ordering
the construction of arcades were the health and safety of the city and its inhabitants; the
promotion of their prosperity; and the improvement of their morals, peace, good order,
comfort, and the convenience. At the time that the ordinance was passed, there was no
national building code enforced to guide the city council; thus, there was no law of national
application that prohibited the city council from regulating the construction of buildings,
arcades and sidewalks in their jurisdiction.
JAIME S. PEREZ vs. SPOUSES MADRONA
G.R. No. 184478
March 21, 2012

FACTS: Fortunito Madrona and Yolanda B. Pante are registered owners of a residential property
located in Greenheights Subdivision, Marikina City. In 1989, respondents built their house
thereon and enclosed it with a concrete fence and steel gate. 1999, respondents received a
letter from petitioner Jaime S. Perez, Chief of the Marikina Demolition Office demanding them
to remove their fence within 7 days. As response, Madrona sent petitioner a three-page letter
stating that the petitioner’s letter (1) contained an accusation libelous in nature as it is
condemning him and his property without due process; (2) has no basis and authority since
there is no court order authorizing him to demolish their structure; (3) cited legal bases which
do not expressly give petitioner authority to demolish; and (4) contained a false accusation
since their fence did not in fact extend to the sidewalk.

Respondents sought the issuance of a TRO and a writ of preliminary injunction to enjoin
petitioner and all persons acting under him from doing any act of demolition on their property
and that after trial, the injunction be made permanent. They also prayed for moral and
exemplary damages and attorney’s fees. Petitioner was served the corresponding summons
and the RTC issued a TRO against petitioner.

ISSUE: Whether or not respondents are entitle to permanent injunction.

RULING: For injunction to issue, two requisites must concur: first, there must be a right to be
protected and second, the acts against which the injunction is to be directed are violative of
said right. Here, the two requisites are clearly present: there is a right to be protected, that is,
respondents’ right over their concrete fence which cannot be removed without due process;
and the act, the summary demolition of the concrete fence, against which the injunction is
directed, would violate said right.

If petitioner indeed found respondents’ fence to have encroached on the sidewalk, his remedy
is not to demolish the same summarily after respondents failed to heed his request to remove
it. Instead, he should go to court and prove respondents’ supposed violations in the
construction of the concrete fence. Indeed, unless a thing is a nuisance per se, it may not be
abated summarily without judicial intervention.
Respondents’ fence is not a nuisance per se. By its nature, it is not injurious to the health or
comfort of the community. It was built primarily to secure the property of respondents and
prevent intruders from entering it. And as correctly pointed out by respondents, the sidewalk
still exists. If petitioner believes that respondents’ fence indeed encroaches on the sidewalk, it
may be so proven in a hearing conducted for that purpose. Not being a nuisance per se, but at
most a nuisance per accidens, its summary abatement without judicial intervention is
unwarranted.
DONATION- EFFECTS and LIMITATIONS

REPUBLIC vs. GUZMAN


G.R. No. 132964
February 18, 2000

FACTS:
David Rey Guzman, a natural-born American citizen, is the son of the spouses Simeon Guzman
(naturalized American) and Helen Meyers Guzman (American citizen). In 1968, Simeon died
leaving to his heirs, Helen and David, an estate consisting of several parcels of land in Bulacan.

Sometime in 1970, Helen and David executed a Deed of Extrajudicial Settlement of the Estate,
dividing and adjudicating to themselves all of the property, and registered it to the RD a year
after. In 1981, Helen executed a Deed of Quitclaim, assigning, transferring and conveying her ½
share of the properties to David. But since it was not registered, she executed another Deed of
Quitclaim to confirm the first.

In 1994, Atty. Batongbacal wrote the OSG andfurnished it with documents showing that
David’s ownership of ½ of the estate was defective. He argued that Art. XII of the Constitution
only allows Filipinos to acquire private lands in the country. The only instances when a foreigner
may acquire private property are by hereditary succession and if he was formerly a natural-
born citizen who lost his Filipino citizenship. Moreover, it contends that the Deeds of Quitclaim
executed by Helen were really donations inter vivos.

ISSUE: Whether or not there was a donation inter vivos.

RULING: NO. Not all the elements of a donation are present. The transfer of the properties by
virtue of a Deed of Quitclaim resulted in the (1) reduction of her patrimony as donor and the (2)
consequent increase in the patrimony of David as donee. However, Helen’s (3) intention to
perform an act of liberality in favor of David was not sufficiently established. The 2 Quitclaims
reveal that Helen intended to convey to her son certain parcels of land and to re-affirm it, she
executed a waiver and renunciation of her rights over these properties. It is clear that Helen
merely contemplated a waiver of her rights, title, and interest over the lands in favor of David,
not a donation. She was also aware that donation was not possible.

Moreover, the essential element of acceptance in the proper form and registration to make
the donation valid is lacking. The SPA executed by David in favor of Atty. Abela was not his
acceptance, but an acknowledgment that David owns the property referred to and that he
authorizes Atty. Abela to sell the same in his name. Further, there was nothing in the SPA to
show that he indeed accepted the donation.

It is also required that the donation be made in a public document and that its acceptance be
made in the same deed of donation or in a separate public document, which has to be recorded
as well.

However, the inexistence of a donation does not make the repudiation of Helen in favor David
valid. There is NO valid repudiation of inheritance as Helen had already accepted her share of
the inheritance when she, together with David, executed a Deed of Extrajudicial Settlement of
the Estate, dividing and adjudicating between them all the properties. By virtue of that
settlement, the properties were registered in their names and for 11 years, they possessed the
land in the concept of owner. Thus, the 2 Quitclaims have no legal force and effect. Helen still
owns ½ of the property.
GONZALO VILLANUEVA vs. SPOUSES BRANOCO
G.R. No. 172804
January 24, 2011

FACTS: Gonzalo Villanueva, represented by his heirs, sued Spouses Branoco to recover a parcel
of land. The former claimed ownership over the property thru purchase from Vere, who in turn,
bought the property from Rodrigo. Gonzalo declared the property in his name for tax purposes
soon after acquiring it. In their answer, the Spouses Baranoco similarly claimed ownership over
the property thru purchase from Rodriguez, who in turn, acquired the property from Rodrigo
byway of donation. The Spouses entered the property and paid taxes afterwards.

The trial court ruled in favor of Gonzalo and declared him owner of the property, and ordered
the Spouses Branoco to surrender possession to Gonzalo. The trial court rejected Spouses
Branoco’s claim of ownership after treating the Deed as a donation mortis causa which Rodrigo
effectively cancelled by selling the Property to Vere. Thus, by the time Rodriguez sold the
property to the Spouses, she had no title to transfer. On appeal, the CA granted the Spouses’
appeal and set aside the trial court's ruling. it held that the deed of donation is one of inter
vivos. In his petition, Gonzalo seeks the reinstatement of the trial court's ruling. Alternatively,
petitioner claims ownership over the Property through acquisitive prescription, having allegedly
occupied it for more than 10 years.

ISSUE: Whether or not the contract between Rodrigo and Rodriguez is a donation or a devise?

RULING: It is immediately apparent that Rodrigo passed naked title to Rodriguez under a
perfected donation inter vivos.

First, Rodrigo stipulated that "if the herein Donee predeceases me, the will not be reverted to
the Donor, but will be inherited by the heirs of Rodriguez," signalling the irrevocability of the
passage of title to Rodriguez's estate, waiving Rodrigo's right to reclaim title. This transfer of
title was perfected the moment Rodrigo learned of Rodriguez's acceptance of the disposition.

Second, What Rodrigo reserved for herself was only the beneficial title to the Property, evident
from Rodriguez's undertaking to "give one [half] of the produce of the land to Apoy Alve during
her lifetime."Thus, the Deed's stipulation that "the ownership shall be vested on [Rodriguez]
upon my demise," taking into account the non-reversion clause, could only refer to Rodrigo's
beneficial title. Indeed, if Rodrigo still retained full ownership over the Property, it was
unnecessary for her to reserve partial usufructuary right over it.

Third, the existence of consideration other than the donor's death, such as the donor's love and
affection to the donee and the services the latter rendered, while also true of devises,
nevertheless "corroborates the express irrevocability of [inter vivos] transfers."

Accordingly, having irrevocably transferred naked title over the Property to Rodriguez in1965,
Rodrigo "cannot afterwards revoke the donation nor dispose of the said property in favor of
another." Thus, Rodrigo's post-donation sale of the Property vested no title to Vere. As Vere's
successor-in-interest, petitioner acquired no better right than him. On the other hand,
respondents bought the Property from Rodriguez, thus acquiring the latter's title which they
may invoke against all adverse claimants, including petitioner.
CENTRAL PHILIPPINES UNIVERSITY vs. COURT OF APPEALS
G.R. No. 112127
July 17, 1995

FACTS: In 1939, the late Don Ramon Lopez was a member of the board of trustees of Central
Philippine University when he executed a donation to the school, stating that the land must be
for exclusive use of a medical college. 50 years later, the heirs of Ramon Lopez filed an action to
annul the donation, stating the failure of the school to construct the medical college over the
land. RTC ruled in favor of respondents, which the CA affirmed.

ISSUE: Whether there is a resolutory condition.

RULING: The donation was an onerous one, where failure of the school to construct a medical
college would give the heirs the power to revoke the donation, reverting the property back to
the heirs of the donor. It is therefore a resolutory condition. Although, the period was not
stated, and the courts should have fixed a period, in this case, 50 years has lapsed since the
donation was executed, thus fixing a period would serve no purpose and the property must
already be reverted back.
INSULAR V EBRADO
G.R. No. L-44059
October 28, 1977

MARTIN, J.:

FACTS:

Cristor Ebrado was issued by The Life Assurance Co., Ltd., a policy for P5,882.00 with a rider for
Accidental Death. He designated Carponia T. Ebrado as the revocable beneficiary in his policy.
He referred to her as hiswife.Cristor was killed when he was hit by a failing branch of a tree.
Insular Life was made liable to pay the coverage in the total amount of P11,745.73,
representing the face value of the policy in the amount of P5,882.00 plus the additional
benefits for accidental death.Carponia T. Ebrado filed with the insurer a claim for the proceeds
as the designated beneficiary therein, although she admited that she and the insured were
merely living as husband and wife without the benefit of marriage.Pascuala Vda. de Ebrado also
filed her claim as the widow of the deceased insured. She asserts that she is the one entitled to
the insurance proceeds.
Insular commenced an action for Interpleader before the trial court as to who should be given
the proceeds. The court declared Carponia as disqualified.

ISSUE:

Whether or not a common-law wife named as beneficiary in the life insurance policy of a legally
married man can claim the proceeds in case of death of the latter?

RULING:

No. Petition denied


Section 50 of the Insurance Act which provides that "the insurance shall be applied exclusively
to the proper interest of the person in whose name it is made.". The word "interest" highly
suggests that the provision refers only to the "insured" and not to the beneficiary, since a
contract of insurance is personal in character. Otherwise, the prohibitory laws against illicit
relationships especially on property and descent will be rendered nugatory, as the same could
easily be circumvented by modes of insurance.
When not otherwise specifically provided for by the Insurance Law, the contract of life
insurance is governed by the general rules of the civil law regulating contracts. And under
Article 2012 of the same Code, any person who is forbidden from receiving any donation under
Article 739 cannot be named beneficiary of a fife insurance policy by the person who cannot
make a donation to him. Common-law spouses are barred from receiving donations from each
other.

Article 739 provides that void donations are those made between persons who were guilty of
adultery or concubinage at the time of donation.There is every reason to hold that the bar in
donations between legitimate spouses and those between illegitimate ones should be enforced
in life insurance policies since the same are based on similar consideration. So long as marriage
remains the threshold of family laws, reason and morality dictate that the impediments
imposed upon married couple should likewise be imposed upon extra-marital relationship.
A conviction for adultery or concubinage isn’t required exacted before the disabilities
mentioned in Article 739 may effectuate. The article says that in the case referred to in No. 1,
the action for declaration of nullity may be brought by the spouse of the donor or donee; and
the guilty of the donee may be proved by preponderance of evidence in the same action.The
underscored clause neatly conveys that no criminal conviction for the offense is a condition
precedent. The law plainly states that the guilt of the party may be proved “in the same acting
for declaration of nullity of donation.” And, it would be sufficient if evidence preponderates.

The insured was married to Pascuala Ebrado with whom she has six legitimate children. He was
also living in with his common-law wife with whom he has two children.

You might also like